You are on page 1of 79

新 GRE 阅读逻辑 10 套

(GRE-RC-Logic-HP-10)
目 录
新 GRE 阅读逻辑 10 套 ......................................................................................................................................................... 1
提示 .................................................................................................................................................................................. 2
练习 1 ............................................................................................................................................................................... 1
练习 2 ............................................................................................................................................................................... 8
练习 3 ............................................................................................................................................................................. 15
练习 4 ............................................................................................................................................................................. 22
练习 5 ............................................................................................................................................................................. 30
练习 6 ............................................................................................................................................................................. 37
练习 7 ............................................................................................................................................................................. 44
练习 8 ............................................................................................................................................................................. 51
练习 9 ............................................................................................................................................................................. 59
练习 10........................................................................................................................................................................... 67
答案 ................................................................................................................................................................................ 76
提示

1. 按目前能够看到的电子材料,新 GRE 考试(Computer-based)每个 Verbal Section 都会出现 1 道逻辑单题,两


个 section 共有 2 道题,内容主要是非学术类的社会生活话题。逻辑单题的文章长度通常在 40-80 字,题
目的五个选项也许比原文更长。这些题目恢复了多年以前古老的 GRE 的逻辑题。在 GMAT 与 LSAT 考试
中,这类题目一直存在,市场上有成熟的应对策略。
2. 我们编选了 95-99 年老 GRE 的逻辑单题作为新 GRE 逻辑单题的训练材料。本材料共汇集 10 个练习,每个
练习 11-15 道题不等。完成时间每道题最好控制在 1-1.5 分钟。建议第一遍时一定卡时间练习。所有 10 个
练习共计 132 道逻辑单题,数量已经足够;反复训练这些题目,应对新 GRE 逻辑单题应该没有困难。
3. 建议进行循环训练:每个练习的题目做 3-4 遍归纳总结以后,再做新的练习。
4. 练习 9 和练习 10 的内容可不做。我们会在完整的新 GRE 阅读训练套题中运用其中的题目,预先不做练习 9
与 10,可避免模拟训练失去真实感。
5. 欢迎同学和朋友们以各种方式对本材料提出改善的建议。
练习 1

1. Patel: Although enrollment in the region's high 2. Peter: More than ever before in Risland, college
school has been decreasing for several graduates with science degrees are accepting permanent
years, enrollment at the elementary school jobs in other fields. That just goes to show that scientists
has grown considerably. Therefore, the in Risland are not being paid enough.
regional school board proposes building a
new elementary school. Lila: No, it does not. These graduates are not
working in science for the simple reason that there are
Quintero: Another solution would be to convert some not enough jobs in science in Risland to employ all of
high school classrooms temporarily into these graduates.
classrooms for elementary school students.
Which of the following, if true in Risland, would most
Which of the following, if true, most helps to support
undermine the reasoning in Peter's argument?
Quintero's alternative proposal?
(A) The college graduates with science degrees who are
(A) Some rooms at the high school cannot be con-
not working in science are currently earning lower
verted into rooms suitable for the use of ele-
salaries than they would earn as scientists.
mentary school students.
(B) Fewer college students than ever before are
(B) The cost of building a high school is higher than
receiving degrees in science.
the cost of building an elementary school.
(C) The number of jobs in science has steadily risen in
(C) Although the birth rate has not increased, the
the last decade.
number of families sending their children to
(D) A significant number of college graduates with
the region's high school has increased markedly.
science degrees worked at low-paying jobs while they
(D) A high school atmosphere could jeopardize the
were in college.
safety and self-confidence of elementary school
(E) Every year some recent college graduates with
students.
science degrees accept permanent jobs in nonscientific
(E) Even before the region's high school population
fields.
began to decrease, several high school class-
rooms rarely needed to be used.

1
3.Counselor: Every year a popular newsmagazine pub- 4. A thorough search of Edgar Allan Poe's correspon-
lishes a list of United States colleges, ranking dence has turned up not a single letter in which he
them according to an overall numerical score mentions his reputed morphine addiction. On the basis
that is a composite of ratings according to sev- of this evidence it is safe to say that Poe's reputation
eral criteria. However, the overall scores gen- for having been a morphine addict is undeserved and
erally should not be used by students as the that reports of his supposed addiction are untrue.
basis for deciding to which colleges to apply.
Which of the following is assumed by the argument
Which of the following, if true, most helps to justify above?
the counselor's recommendation? (A) Reports claiming that Poe was addicted to mor-
(A) The vast majority of people who purchase the phine did not begin to circulate until after his
magazine in which the list appears are not death.
college-bound students. (B) None of the reports of Poe's supposed morphine
(B) Colleges that are ranked highest in the magazine's addiction can be traced to individuals who actu-
list use this fact in advertisements aimed at ally knew Poe.
attracting students. (C) Poe's income from writing would not have been
(C) The rankings seldom change from one year to sufficient to support a morphine addiction.
the next. (D) Poe would have been unable to carry on an
(D) The significance that particular criteria have for extensive correspondence while under the
any two students is likely to differ according to influence of morphine.
the students' differing needs. (E) Fear of the consequences would not have pre-
(E) Some college students who are pleased with their vented Poe from indicating in his correspon-
schools considered the magazine's rankings dence that he was addicted to morphine.
before deciding which college to attend.

2
5. Adelle: The government's program to reduce the Which of the following, if true, most strongly counters
unemployment rate in the province of Carthena by Fran's objection to Adelle's argument?
encouraging job creation has failed, since the rate there (A) The government is advised by expert economists,
has not changed appreciably since the program began a some of whom specialize in employment issues.
year ago. (B) The unemployment rate in the province of Carthena
Fran: But the unemployment rate in Carthena had has historically been higher than that of the country as a
been rising for three years before the program began, so whole.
the program is helping. (C) The current government was elected by a wide
margin, because of its promises to reduce the
unemployment rate in Carthena.
(D) Around the time the government program began,
large numbers of unemployed Carthena residents began
leaving the province to look for work elsewhere.
(E) The unemployment rate in Carthena had been
relatively stable until shortly before the current
government took office.

3
6. Soft Drink Manufacturer:Our new children's soft 7.Over a period of several months, researchers attached
drink, RipeCal, is fortified with calcium. small lights to the backs of wetas—flightless insects
Since calcium is essential for developing native to New Zealand—enabling researchers for the
healthy bones, drinking RipeCal regularly first time to make comprehensive observations of the
will help make children healthy. insects' nighttime activities.Thus, since wetas forage
only at night, the researchers' observations will
Consumer Advocate:But RipeCal also contains large significantly improve knowledge of the normal
amounts of sugar, and regularly consuming foraging habits of wetas.
large amounts of sugar is unhealthful, especially
for children. Which of the following is an assumption on which
the argument depends?
In responding to the soft drink manufacturer, the (A) Researchers were interested only in observing
consumer advocate does which of the following? the wetas' foraging habits and so did not keep
(A)Challenges the manufacturer's claim about the track of other types of behavior.
nutritional value of calcium in children's diets (B) No pattern of behavior that is exhibited by wetas
(B)Argues that the evidence cited by the manufac- during the nighttime is also exhibited by wetas
turer, when properly considered, leads to a during the daytime.
conclusion opposite to that reached by the (C)Attaching the small lights to the wetas' backs
manufacturer. did not greatly alter the wetas' normal night-
(C)Implies that the manufacturer of a product is time foraging habits.
typically unconcerned with the nutritional value (D)Wetas typically forage more frequently during
of that product. the months in which the researchers studied
(D)Questions whether a substance that is healthful them than they do at other times.
when eaten in moderation can be unhealthful (E)The researchers did not use other observational
when eaten in excessive amounts. techniques to supplement their method of using
(E)Presents additional facts that call into question small lights to track the nighttime behavior of
the conclusion drawn by the manufacturer. wetas.

4
8.People whose bodies cannot produce the substance 9.The early universe contained only the lightest
cytochrome P450 are three times as likely to develop elements, hydrogen and helium. Heavier elements,
Parkinson's disease, a disease that affects the brain, such as carbon, form only in nuclear reactions in
as are people whose bodies do produce this substance. stars and are dispersed when the stars explode. A
Since cytochrome P450 protects the brain from toxic recently discovered gas cloud contained carbon
chemicals, toxic chemicals probably play a role in the several billion years ago, when the universe was
development of Parkinson's disease. no more than two billion years old.

Which of the following, if true, most strongly If the statements above are true, which of the
supports the argument? following must, on the basis of them, also be true?
(A)It will soon be possible for cytochrome P450 to (A)The earliest stars contained only hydrogen.
be synthesized for the treatment of people (B)Some stars were formed before the universe was
whose bodies cannot produce this substance. two billion years old.
(B)Many people whose bodies are unable to (C)The carbon in the gas cloud later formed part of
produce cytochrome P450 lack the ability to some stars.
produce certain other substances as well. (D)No stars identified to date are as old as the gas
(C)Cytochrome P450 has no effect on the brain cloud.
other than to protect it from toxic chemicals. (E)The gas cloud also contained hydrogen and
(D)People with Parkinson's disease often exhibit a helium.
marked lessening in the severity of their symp-
toms when they are treated with dopamine, a
chemical produced naturally in the brain.
(E)Many people with Parkinson's disease have the
ability to produce cytochrome P450 naturally.

5
10.Sleep deprivation is a known cause of workplace 11.A list of the fifteen operas most frequently performed
error, and many physicians frequently go without in recent times includes no works by the nineteenth-
sleep for periods of 24 hours or more. However, few century German composer Richard Wagner. Although
of these physicians have, in the course of a routine music producers tend to produce what audiences want,
examination by a peer, been diagnosed with sleep relative infrequency of performance probably does
deprivation.So there is little cause for concern that not indicate lack of popularity in Wagner's case, since
habitual sleep deprivation will cause widespread Wagner's operas are notoriously expensive to perform
physician error. on stage.

The answer to which of the following questions Which of the following, if true, most strongly
would be most helpful in evaluating the argument? supports the conclusion of the argument above?
(A)Do physicians who have been diagnosed with (A)The list of most frequently performed operas
sleep disorders also show signs of other ills not does not include operas produced by small
related to sleep deprivation? amateur groups.
(B)Is the ability to recognize the symptoms of sleep (B)Some opera companies are backed by patrons
deprivation in others significantly impaired by who are willing to commit large sums of
habitual sleep deprivation? money in order to enjoy lavish productions.
(C)Do factors other than habitual sleep deprivation (C)All of the fifteen most frequently performed
ever lead to errors in the workplace on the part operas of recent times are works that have
of physicians? been popular for at least 75 years.
(D)Of people who have recently been treated by (D)More recordings have been produced recently of
physicians, what percentage believe that many the works of Wagner than of the works of any
physicians have occasionally suffered from sleep other composer of opera.
deprivation? (E)Operatic works of all kinds have been increasing
(E)Is the incidence of sleep deprivation higher in popularity in recent years.
among physicians than it is among other health
care workers?

6
12.The bodies of dwarf individuals of mammalian 13.Excluding purchases by businesses, the average
species are generally smaller in relation to those of amount spent on a factory-new car has risen 30 per-
nondwarf individuals than are the teeth of the dwarf cent in the last five years. In the average household
individuals in relation to those of the nondwarf indi- budget, the proportion spent on car purchases has
viduals. Fragmentary skeletal remains of an adult remained unchanged in that period. Therefore the
dwarf woolly mammoth were recently found. The teeth average household budget must have increased by
are three-fourths the size of the teeth of an average 30 percent over the last five years.
adult nondwarf woolly mammoth.
Which of the following is an assumption on which
The statements above, if true, most strongly support the argument relies?
which of the following? (A)The average number of factory-new cars pur-
(A)The body of the dwarf woolly mammoth was chased per household has remained unchanged
less than three-fourths the size of the body of over the last five years.
an average adult nondwarf woolly mammoth. (B)The average amount spent per car by businesses
(B)None of the teeth of the dwarf woolly mammoth buying factory-new cars has risen 30 percent in
that were recently discovered was as large as the last five years.
any of the teeth of nondwarf woolly mammoths (C)The proportion of the average household budget
that have been discovered. spent on all car-related expenses has remained
(C)The teeth of most adult dwarf individuals of unchanged over the last five years.
mammalian species are three- fourths the size of (D)The proportion of the average household budget
the teeth of the adult nondwarf individuals of spent on food and housing has remained
the same species. unchanged over the last five years.
(D)Dwarf woolly mammoths had the same number (E)The total amount spent nationwide on factory-
of teeth as did nondwarf woolly mammoths. new cars has increased by 30 percent over the
(E)Dwarf individuals of most mammalian species last five years.
are generally no more than three-fourths the
size of the adult nondwarf individuals of those
species.

7
练习 2

1. Armtech, a temporary-employment agency, previously 2 The global population of frogs has declined in recent
gave its employees 2.5 paid vacation days after each years while the amount of ultraviolet radiation reaching
700 hours worked. Armtech's new policy is to give the Earth has increased. Since the genetic material in
its employees 5.0 paid vacation days after each 1,200 frog eggs is harmed when exposed to ultraviolet radi-
hours worked. Therefore, this new policy is more ation, and since the eggs themselves are not protected
generous to Armtech employees in giving them more by shells or leathery coverings but are gelatinous, the
vacation days per hour worked than the old policy did. frog population decline is probably due, at least in
part, to the ultraviolet radiation increase.
Which of the following is an assumption on which
the argument depends? Which of the following, if true, provides the strongest
(A) Most current Armtech employees approve of the support for the argument?
company's new vacation policy. (A) Even in those regions where there has been no
(B) A few Armtech employees leave the company significant increase in ultraviolet radiation, only
before having worked 700 hours. a small proportion of the frog eggs that are laid
(C) Most Armtech employees were not aware that ever hatch.
the company planned to change its vacation (B) In areas where there has been the least decline
policy until after it had already done so. in frog populations, populations of species of
(D) A significant portion of Armtech employees stay insects that frogs eat have decreased.
with the company long enough to work for (C) The eggs of frog species whose populations are
1,200 hours. declining tend to have higher concentrations of
(E) Armtech's new vacation policy closely matches damaging pesticides than do the eggs of frog
the vacation policies of competing temporary species whose populations have not declined.
employment agencies. (D) In many places where turtles, which lay eggs
with tough, leathery coverings, share habitats
with frogs, turtle populations are also in decline.
(E) Populations of frog species that hide their eggs
beneath rocks or under sand have declined
considerably less than have populations of frog
species that do not cover their eggs.

8
3 To improve productivity, manufacturing companies 4.During the nineteenth century, Britain's urban popu-
have recently begun restructuring work to produce lation increased as its rural population diminished. A
more goods with fewer assembly-line workers, and historian theorizes that, rather than industrialization's
the companies have laid off many workers as a being the cause, this change resulted from a series
consequence. The workers laid off have been those of migrations to urban areas, each occasioned by a
with the least seniority(time on the job), generally depression in the agrarian economy. To test this hypoth-
the younger workers. esis, the historian will compare economic data with
population census data.
The statements above, if true, most strongly support
which of the following as a conclusion? The historian's hypothesis would be most strongly
(A) The products manufactured by the companies are supported if which of the following were found to be
not undergoing design changes while the true?
manufacturing jobs are being restructured. (A) The periods of greatest growth in the industrial
(B) When assembly-line workers have made sug- economy were associated with a relatively
gestions for improvements in manufacturing rapid decline in the rural population.
processes, some suggestions have been (B) The periods of greatest weakness in the agrarian
implemented, but many have not. economy were associated with relatively slow
(C) Assembly-line workers now need increased growth in the population as a whole.
reading and mathematical skills to do their (C) Periods when the agrarian economy was compar-
jobs. atively strong and the industrial economy com-
(D) Some of the innovations in assembly-line paratively weak were associated with a particu-
processes and procedures that were made to larly rapid decline in the rural population.
increase productivity have instead proved to be (D) Periods when the agrarian and industrial econo-
counterproductive. mies were both strong were associated with
(E) The manufacturing companies are increasing the particularly rapid growth in the urban popula-
average age of their assembly-line workforce tion.
while still seeking to increase production. (E) The periods of greatest strength in the agrarian
economy were associated with relatively slow
growth in the urban population.

9
5 Politician:Each year, small businesses create more 6 In the workplace, influenza is typically spread by
jobs than do large established businesses. Therefore, in infected individuals to others with whom they work
order to reduce unemployment in the long term, we in close quarters. A new medication that suppresses
should provide incentives for starting small businesses the symptoms of influenza therefore will actually
rather than for expanding established large businesses. increase the number of influenza cases, because this
medication will allow people who would otherwise be
Which of the following, if true, casts the most doubt home in bed to return to work while infected.
on the politician's argument?
(A) In general, people employed by small businesses Which of the following, if true, most seriously chal-
report higher job satisfaction than do people lenges the prediction?
employed by large businesses. (A) Coughing, a symptom of influenza that the new
(B) Among the currently unemployed are many medication suppresses, is a primary mechanism
people with sufficient job skills to perform the in the spread of this illness.
jobs that small businesses would create. (B) Some medications that are used to suppress
(C) Providing an effective incentive for starting a symptoms of influenza are also used by many
business generally costs significantly less than people to treat symptoms that are caused not
providing an effective incentive for expanding by influenza but by other illnesses.
a large business. (C) Many workers who now remain at home when
(D) A high proportion of small businesses fail within infected with influenza do so because the
three years of starting because of their owners' symptoms of influenza prevent them from per-
inexperience. forming their jobs effectively.
(E) The average large business contributes more (D) Most adults who are immunized against influ-
money to politicians’ campaign funds than the enza in order to avoid being infected are over
average small business does. 65 years old and retired and thus do not work
outside the home.
(E) Symptoms of an illness are often the body's
means of curing itself of the illness, and there-
fore suppression of symptoms can prolong the
illness that causes them.

10
7 Editorial: Which of the following is an assumption on which the
Critics of nuclear power complain about the allegedly argument depends?
serious harm that might result from continued operation (A) Existing nuclear power plants should be closed only
of existing nuclear power plants. But such concerns do if it can be conclusively demonstrated that their
not justify closing these plants;after all, their operation continued operation is likely to cause harm more
has caused no more harm than that caused by pollution serious than the harm their operation has already
generated by coal-and oil-burning power plants, the caused.
most important other sources of energy. (B) Closing existing nuclear power plants would require
greatly increased reliance on coal-and oil-burning
power plants.
(C) The harm that has resulted from operation of
existing coal-and oil-burning power plants has
been significant.
(D) The harm that a nuclear power plant is likely to
cause as it continues to operate can be reliably
predicted from the past history of nuclear power
plants.
(E) The only harm that has resulted from operation of
existing coal-and oil-burning power plants has
resulted from the pollution generated by these
plants.

11
8 Drug manufacturer: Although our company requires 9 Virginia and her brother William disagree over when
that patients who use our new drug also pur- their father was born: Virginia claims it was in 1935
chase from us nonreusable kits for weekly and William claims it was in 1933. The hospital
blood testing, the expense of those kits is an where their father was born has no records for 1933
entirely necessary one: weekly blood testing but has complete records for 1935—records that do
must be done to monitor the drug's potential not include a birth record for their father. Therefore,
side effects, which can be very dangerous. he must have been born in 1933.

Which of the following, if true, most seriously weak- The argument depends on which of the following
ens the manufacturer's argument? assumptions?
(A) The expense of purchasing the blood-test kits (A) Either Virginia's claim or William's claim is
has not prevented any patients from obtaining correct.
them or the drug. (B) The records of the hospital where their father
(B) Medical laboratories can perform the blood test- was born date back to 1933.
ing at a lower cost to patients or their insurers (C) Virginia and William know the day and the
than the price the manufacturer charges for the month of their father's birth.
kits. (D) There are urgent practical reasons why Virginia
(C) A one-year supply of the drug and the weekly and William must know the date of their
blood-test kits can cost patients or their father's birth.
insurers over $10,000. (E) None of their other relatives knows the year in
(D) Most government and other health insurance which Virginia and William's father was born.
programs will not reimburse patients for the
full cost of both the drug and the blood-test
kits.
(E) Patients who suffer one or more of the danger-
ous side effects of the drug can incur heavy
expenses for the treatment of those side effects.

12
10 The town of San Leonardo has recently enacted a law 11. Children whose biological parents both have Tic
banning smoking in all restaurants within town limits. Syndrome Z (TSZ), which is characterized by the
Since many smokers who normally dine in San involuntary contraction of certain muscles, are about
Leonardo's restaurants will not want to refrain from four times more likely to develop such contractions
smoking during their meals, San Leonardo's restau- than are children whose biological parents do not
rants will undoubtedly lose many patrons and con- have TSZ, It is likely, therefore, that predisposition
siderable income. to TSZ is an inherited trait.

Which of the following, if true, most helps to Which of the following, if true, would most strengthen
strengthen the argument above? the conclusion above?
(A) Most residents of San Leonardo who eat in res- (A) Children whose parents have TSZ are more
taurants are not smokers. likely to develop TSZ if they are under unusual
(B) Most smokers who dine in the company of non- stress at school or at home than if they are not
smokers are willing to refrain from smoking under such stress.
during their meals. (B) Children whose biological parents do not have
(C) If the law banning smoking in restaurants had TSZ are more likely to develop TSZ if they are
not been enacted, it is likely that a more raised by adoptive parents with TSZ than if
stringent law banning smoking in all public they are raised by their biological parents.
places in San Leonardo would have been (C) Children whose biological parents have TSZ are
enacted instead. as likely to develop TSZ if they are raised by
(D) Prior to the enactment of the law banning adoptive parents who do not have TSZ as if
smoking in San Leonardo's restaurant, the they are raised by their biological parents.
town had a law that required most restaurants (D) Children whose biological parents have TSZ and
to have nonsmoking sections. who develop TSZ usually avoid developing a
(E) None of the other communities adjacent to San severe form of the syndrome if they seek
Leonardo, which have restaurants comparable treatment for TSZ shortly after developing the
to those of San Leonardo, has enacted and first signs of it.
enforces any antismoking legislation. (E) Children with TSZ whose biological parents do
not have TSZ are less likely to have the syn-
drome diagnosed when symptoms first appear
than are children with TSZ whose biological
parents have TSZ.

13
12. Playing eighteenth-century music on the instruments If the statements above are true, which of the
of that period provides valuable information about how following must be true on the basis of them?
the music originally sounded. Eighteenth-century (A) Eighteenth-century instruments cannot be used to
instruments cannot be played without being restored, provide information about the original techniques used
however, and restoring such an instrument destroys all of in playing such instruments if they have been restored.
the information that researchers could obtain from it (B) Eighteenth-century instruments that have been
about eighteenth-century instrument-making techniques. restored can provide information only about how
eighteenth-century music originally sounded
(C) Eighteenth-century instruments are the only
source of information about the instrument-making
techniques of that period.
(D) An eighteenth-century instrument that has not
been restored can provide more information than can
one that has been restored.
(E) An eighteenth-century instrument cannot serve as
a source of new information about eighteenth-century
instrument-making techniques once it can be played

14
练习 3

1.A new and more aggressive form of the fungus that 2.As government agencies, faced with budget difficul-
caused the Irish potato famine of the nineteenth century ties, reduce their funding for scientific research, a
has recently arisen. However, since this new form of the greater amount of such research is being funded by
fungus can be killed by increased application of private foundations. This shift means that research
currently used fungicides, it is unlikely that the fungus projects likely to produce controversial results will
will cause widespread food shortages in countries that almost certainly comprise a smaller proportion of all
currently rely on potatoes for sustenance. funded research projects, since private foundations,
concerned about their public image, tend to avoid
Which of the following, if true, most calls into question controversy.
the conclusion in the argument above?
(A) Though potatoes are an important staple crop in Which of the following is an assumption on which
many parts of the world, people in most countries rely the argument depends?
primarily on wheat or rice for sustenance. (A) Only research that is conducted without concern
(B) Potato farmers in many countries to which the new for the possibility of generating controversy is
form of the fungus has spread cannot afford to increase likely to produce scientifically valid results.
their spending on fungicides. (B) Private foundations that fund scientific research
(C) The new form of the fungus first began to spread projects usually recognize that controversial
when contaminated potato seeds were inadvertently results from those projects cannot always be
exported from a major potato-exporting country. avoided.
(D) Potato farmers in most countries use several (C) Scientists who conduct research projects funded
insecticides on their crops in addition to fungicides of by private foundations are unlikely to allow the
the sort that kill the new form of the fungus. concerns of the funding organizations to influ-
(E) Most governments have funds set aside that can be ence the manner in which they conduct the
used to alleviate the effects of large-scale disasters such research.
as severe food shortages and floods. (D) Many government agencies are more concerned
about their public image than are most private
foundations.
(E) Government agencies are more willing than are
private foundations to fund research projects
that are likely to produce controversial results.

15
3.Juries in criminal trials do not base verdicts on uncor- 4.Although spinach is rich in calcium, it also contains
roborated testimony given by any one witness. Rightly large amounts of oxalic acid, a substance that greatly
so, because it is usually prudent to be highly skeptical impedes calcium absorption by the body. Therefore,
of unsubstantiated claims made by any one person. other calcium-containing foods must be eaten either
But then, to be consistent, juries should end an all- instead of or in addition to spinach if a person is to
too-common practice: convicting defendants on the be sure of getting enough calcium.
basis of an uncorroborated full confession.
Which of the following, if true, most strengthens the Which of the following, if true, most seriously
argument above? weakens the argu-ment above?
(A) Juries often acquit in cases in which a defendant (A)Rice, which does not contain calcium,
retracts a full confession made before trial. counteracts the effects of oxalic acid on
(B) The process of jury selection is designed to screen calcium absorption.
out people who have a firm opinion about the (B) Dairy products, which contain even more
defendant's guilt in advance of the trial. calcium than spinach does, are often eaten by
(C) Defendants sometimes make full confessions when people who eat spinach on a regularbasis.
they did in fact do what they are accused of (C) Neither the calcium nor the oxalic acid in
doing and have come to believe that the prose- spinach is destroyed when spinach is cooked.
cutor has compelling proof of this. (D) Many leafy green vegetables other than spinach
(D) Highly suggestible people who are accused of that are rich in calcium also contain high
wrongdoing sometimes become so unsure of concentrations of oxalic acid.
their own recollection of the past that they can (E) Oxalic acid has little effect on the body's ability to
come to accept the accusations made against absorb nutrients other than calcium.
them.
(E) Many people believe that juries should not con-
vict defendants who have not made a full con-
fession.

16
5.Joyce: Three years ago the traffic commission 6. Women make up the majority of the population in the
modified our town's busiest intersection for country, and many of the prescriptions written by
better visibility, a commendable effort to cut doctors for tranquilizers are for women patients. The
down on traffic accidents there. testing of these drugs for efficacy and the calibration
of recommended doses, however, was done only on
Gary: Over the past three years there have been men. Not even the animals used to test toxicity were
more, not fewer, traffic accidents per week at female.
that intersection, so the modification has
increased the likelihood of accidents there The statements above, if true, best support which of
the following as a conclusion?
The answer to which of the following questions (A) Some tranquilizers are more appropriately pre-
would be most useful in evaluating Gary's argument? scribed for women than for men.
(A) What proportion of the town's drivers involved (B) There have been no reports of negative side
in accidents that occurred prior to the effects from prescribed tranquilizers in women
modification suffered personal injury in their (C) Tranquilizers are prescribed for patients in some
accidents? instances when doctors do not feel confident of
(B) How long, on average, had the members of the their diagnoses.
traffic commission held their offices when the (D) The toxicity of drugs to women is less than the
modification was implemented? toxicity of the same drugs to men.
(C) Do a majority of the town's residents approve of (E) Whether the recommended dosages of tranquil-
the traffic commission's overall performance? izers are optimal for women is not known.
(D) What measures have nearby towns taken within
the last three years in order to improve visibil-
ity at dangerous intersections?
(E) How has the volume of traffic at the town's
busiest intersection changed over the last three
years?

17
7. Gray wolves have been absent from a large national 8. Osteoporosis is a disease that reduces bone mass, lead-
park for decades. Park officials wish to reestablish ing to fragile bones that break easily. Current treat-
the wolves without jeopardizing any existing species ments for osteoporosis such as estrogen or calcitonin
of wildlife there. Since the park contains adequate help prevent further loss of bone but do not increase
prey for the wolves and since the wolves avoid close bone mass. Since fluoride is known to increase bone
contact with people, reintroducing them would serve mass, administering fluoride to osteoporosis patients
the officials' purpose without seriously jeopardizing would therefore help make their bones less susceptible
visitors' safety. to breaking.

Each of the following, if true, strengthens the Which of the following, if true, most seriously
argument above EXCEPT: weakens the argument above?
(A) The park is so large that wolves will not need to (A) Most people who suffer from osteoporosis are
venture into areas frequented by people. not aware that administration of fluoride can
(B) Rabies is very rare in wolves, and there have increase bone mass.
been no verified cases of serious human injuries (B) Fluoride is added to drinking water in many
from nonrabid wild wolves since records have locations in order to strengthen the teeth of
been kept. people who drink the water.
(C) Ranchers in the region near the park have (C) The risk of contracting osteoporosis and other
expressed concern that gray wolves, if reintro- degenerative bone diseases is lessened by exer-
duced, would sometimes prey on their livestock. cise and an adequate intake of calcium.
(D) Predation by gray wolves on elk in the park is (D) Unlike administration of fluoride, administration
likely to improve the health and viability of the of estrogen or calcitonin is known to cause
park's elk population as a whole by reducing undesirable side effects for many people.
malnutrition among the elk. (E) The new bone mass that is added by the admin-
(E) Wolves do not prey on animals of any endangered istration of fluoride is more brittle and less
species that currently inhabit the park. elastic than normal bone tissue.

18
9. The closest distance from which an asteroid has been Which of the following can be properly concluded
photographed using ground-based radar is 2.2 million from the statements above?
miles, the distance from which the asteroid Toutatis (A) Toutatis is more likely to collide with the Earth
was recently photographed. The closest photograph of than Gaspra is.
an asteroid is of Gaspra, which was photographed (B) Toutatis, unlike Gaspra, has only recently been
from a distance of only 10,000 miles. discovered.
(C) Asteroids can be photographed only by using
ground-based radar.
(D) Ground-based radar photography cannot take
photographs of objects much beyond 2.2 million
miles from Earth.
(E) The photograph of Gaspra was not taken using
ground-based radar.

19
10. Which of the following most logically completes the 11. Fossils of the coral Acrocora palmata that date from
argument below? the last period when glaciers grew and consequently
Alone among living species, human beings spread from the polar regions are found at ocean
experience adolescence, a period of accelerated physical depths far greater than those at which A. palmata can
growth prior to full maturity. Whether other hominid now survive. Therefore, although the fossilized
species, which are now all extinct and are known only A. palmata appears indistinguishable from A. palmata
through the fossil record, went through adolescence now living, it must have differed in important respects
cannot be known, since to have been able to live in deep water.
(A) the minimum acceleration in physical growth
The argument depends on the assumption that
that would indicate adolescence might differ
(A) no fossils of the coral A. palmata have been
according to species
found that date from periods when glaciers
(B) the fossil record, though steadily expanding, will
were not spreading from the polar regions
always remain incomplete
(B) geological disturbances since the last period
(C) detecting the adolescent growth spurt requires
during which glaciers spread have caused no
measurements on the same individual at differ-
major downward shift in the location of
ent ages
A. palmata fossils
(D) complete skeletons of extinct hominids are
(C) A. palmata now live in shallow waters in most
extremely rare
of the same geographical regions as those in
(E) human beings might be the first species to bene-
which deep-lying A. palmata fossils have been
fit from the survival advantages, if any, con-
found
ferred by adolescence
(D) A. palmata fossils have been found that date
from each of the periods during which glaciers
are known to have spread from the polar region
(E) A. palmata can live at greater depths where the
ocean temperature is colder than they can
where the ocean temperature is warmer

20
12.Conservationists have believed that by concentrating Which of the following findings about widely scattered
their preservation efforts on habitats rich in an easily tracts 10 kilometers by 10 kilometers in a temperate
surveyed group of species, such as birds, they would climate zone would most seriously challenge the
thereby be preserving areas rich in overall species conservationists' assumptions?
diversity. This belief rests on a view that a geographical
area rich in one group of species will also be rich in the (A) The tracts show little damage from human intrusion
other groups characteristic of the entire regional climate and from pollution by human activities.
zone. (B) Where a certain group of species, such as birds, is
abundant, there is also an abundance of the species, such
as insects, on which that group of species feeds, or in
the case of plants, of the land and water resources it
requires.
(C) The area of one of the tracts is generally large
enough to contain a representative sample of the
organisms in the region.
(D) There is little overlap between the list of tracts that
are rich in species of butterflies and the list of those that
are rich in species of birds.
(E) The highest concentration of individuals of rare
species is found where the general diversity of species is
greatest.

21
练习 4

1. Drug companies lose money when manufacturing 2.The tomb of a warrior killed in 1501 bears a sculpted
drugs that cure those suffering from rare diseases portrait depicting him dressed for battle.Some his-
because selling a drug to only a few people usually torians attribute the portrait to an artist from that
does not recoup manufacturing expenses.Therefore, century, but of the many references to the tomb in
a company manufacturing any of the drugs that cure surviving documents, none that predates the 1800's
those suffering from loxemia, an extremely rare mentions the portrait.The portrait is therefore more
disease, will undoubtedly lose money. likely the work of a much later artist.

Which of the following, if true, most seriously Which of the following, if true, would also support
weakens the conclusion above? the conclusion of the argument if substituted for the
(A)Several drugs that cure those suffering from evidence given concerning the portrait?
loxemia also cure those suffering from very (A)The portrait of the warrior was commissioned by
common illnesses. the family of the warrior's widow.
(B)Most of those who contract loxemia also con- (B)References in surviving documents mention that
tract another illness concurrently. an artist was paid in 1525 for an unspecified
(C)Most of the drug companies that manufacture number of works for the church in which the
drugs that cure rare diseases do not manufac- tomb is located
ture drugs that cure loxemia. (C)The warrior is depicted in the portrait as wearing
(D)A sizable number of people are afflicted with boots made of a material not used for boots
one or another rare disease even though each until the 1700's.
rare disease afflicts only a small number of (D)Some other art treasures from the church in
people. which the tomb is located have been reliable
(E)The larger the amount of a drug that is manu- dated to the 1400's.
factured, the lower the manufacturing expense (E)The portrait of the warrior on the tomb strongly
for each unit of the drug that is produced. resembles a portrait of him known to have
been completed during his lifetime.

22
3.Scientist:More than 1, 000 large asteroids regularly 4.It has long been thought that high levels of the hor-
cross the Earth's path.Even though the probabil- mone testosterone contribute to the onset of heart
ity of one colliding with the Earth is extremely disease in men.However, this view cannot be correct,
slight, we should do whatever we can to reduce since men who have heart disease typically show
that probability since any such collision would significantly lower levels of testosterone than do men
be catastrophic.The best way to avoid such a who have not had heart disease.
disaster is to deflect the asteroids.The only
known way of deflecting asteroids is by hitting The argument above assumes which of the following?
them with nuclear weapons that would be stored (A)Many men who have never had heart disease
in space stations. have unusually low levels of testosterone.
(B)Having heart disease does not significantly lower
The scientist’s claims are structured so as to lead to the level of testosterone in men.
which of the following conclusions? (C)Levels of hormones other than testosterone
(A)Nuclear technology is the only technology that significantly affect the likelihood that a man
can plausibly be used to prevent natural will develop heart disease.
catastrophes. (D)Heart disease and lowered testosterone levels in
(B)Nuclear weapons should be deployed in space. men are the effects of a single common cause.
(C)No catastrophe has yet been caused by the (E)High levels of testosterone have never been
collision of an asteroid with the Earth. thought to contribute to a serious disease other
(D)The 1, 000 large asteroids that cross the Earth's than heart disease.
path pose only an extremely slight risk of
colliding with the Earth.
(E)There is currently no acceptable use to which
nuclear weapons can be put, aside from pro-
tecting the Earth from asteroids.

23
5.People who engage in scuba diving are healthier, on The argument is most vulnerable to criticism on the
average, than people who do not engage in this grounds that it
activity.Therefore, scuba diving tends to promote (A)presupposes that everyone who takes up scuba
improved health. diving does so solely for health reasons
(B)leads to a further and falsifiable conclusion that
no one can achieve good health without
engaging in scuba diving
(C)fails to point out that a small number of people
are seriously injured in scuba diving accidents
each year
(D)treats a precondition for improving one's health
as though it were something that by itself
could ensure good health
(E)overlooks the possibility that people generally do
not take up scuba diving unless they are in
good health

24
6.Which of the following most logically completes the 7.In Bassaria a group of that country's most senior
argument below? judges has criticized the uniform mandatory sentences
recently introduced for certain specific crimes.The
In recent years, the proportion of car buyers who buy judges argue that such sentences, by depriving them
new cars rather than used cars has declined.Some of all discretion in setting sentences, make it impos-
consumers have attributed this change to an increase sible for them to consider either aggravating or exten-
in new-car prices.As evidence of the price increase, uating circumstances and so make it impossible to
they cite figures that show that, even adjusting for achieve true justice―the fitting of the severity of the
inflation, the price that the buyer of a new car pays, punishment to the gravity of the particular crime.
on average, is far higher now than a few years ago.
This evidence is unpersuasive, however, because Which of the following, if true, provides the strongest
(A)the value of a car that is bought new declines evidence for the claim that in Bassaria the newly
much more rapidly than does the value of a introduced mandatory sentences are not necessarily a
car that is bought used change for the worse with respect to achieving true
(B)after someone has bought a car, it might be justice as defined in the argument?
several years before that person next buys (A)Before mandatory sentencing, judges in eastern
a car Bassaria imposed strikingly different sentences
(C)a decline in the proportion of car buyers who from those in western Bassaria for equally
buy new cars must necessarily mean that the grave instances of the same kind of offense.
proportion who buy used cars has increased (B)In Bassaria the frequency of crimes that have
(D)the relative increase in used-car sales might be been made subject to mandatory sentences is
explained by the decisions of only a small lower now than it was just prior to the intro-
proportion of all car buyers duction of mandatory sentencing.
(E)the change in the average price paid for a new (C)The law introducing mandatory sentences was
car could result solely from more people's passed in the legislature of Bassaria by a large
rejecting inexpensive new cars in favor of used majority and is unlikely to be repealed in the
cars foreseeable future.
(D)There used to be a wide difference between the
minimum and the maximum sentences allowed
by law in cases of crimes now subject to man-
datory sentences.
(E)In Bassaria judges are appointed for life and are
thus not easily influenced by political pressure
groups.

25
8.Each of two particular inspection systems that are based Which of the following most precisely characterizes the
on different principles would detect all product flaws but reasoning in the argument?
would also erroneously reject three percent of flawless (A)The reasoning is conclusive, that is, the conclusion
products.Assuming there is no overlap between the cannot be false if the statements offered in its
products erroneously rejected by the two systems and support are true.
also no interference between the systems if both operate, (B)The reasoning is strong but not conclusive, if the
using both systems and rejecting only those products statements offered in support of the conclusion are
found flawed by both would be a way of avoiding all true, they provide good grounds for that conclu-
erroneous rejections. sion, though it is possible that additional infor-
mation might weaken the argument.
(C)The reasoning is weak; the statements offered in
support of the conclusion, though relevant to it,
by themselves provide at best inadequate grounds for
the conclusion.
(D)The reasoning is flawed in that the conclusion is no
more than a paraphrase of one of the pieces of
evidence offered in its support.
(E)The reasoning is flawed in that the argument treats
evidence that a factor is necessary to bring about
an event as if it were evidence that the factor is
sufficient to bring about that event.

26
9. In recent years, there has been a dramatic decline in 10. Census data for Prenland show that unmarried
the population of the shrike, a predatory bird that Prenlandic men in their thirties outnumber unmarried
inhabits flat land, such as farms and pastures. Some Prenlandic women in that age group by about ten to
ornithologists hypothesize that this decline is due to One. Most of these men do wish to marry. Clearly,
the introduction of new, more effective pesticides to however, unless many of them marry women who
control the insect species on which shrikes prey. are not Prenlandic, all but a minority will remain
unmarried.
The answer to which of the following questions is
NOT relevant to evaluating the ornithologists' The argument makes which of the following assump-
hypothesis? tions?
(A) Was there a decline in the shrike population (A) Emigration from Preland is more common
before the new pesticides were first used? among women than among men.
(B) Have shrike populations declined significantly (B) A greater proportion of Prelandic women
in those habitats where the new pesticides have in their thirties than of Prenlandic men of the
not been used? same age would prefer to remain unmarried.
(C) Have the new pesticides more significantly (C) It is unlikely that many of these unmarried
reduced the population of insect species on Prenlandic men will marry women more than
which shrikes prey than did the pesticides a few years older than themselves.
previously (D) Prenland has a high rate of divorce.
used? (E) Most of the unmarried Prenlandic men are
(D) Are insects that have consumed the new pesti- unwilling to marry women who are not
cides more toxic to the shrikes that eat those Prenlandic.
insects than were insects that consumed the
less effective pesticides?
(E) Are the new pesticides considered by most
people to be less harmful to the environment
than the old pesticides were considered to be?

27
11.Certain extremely harmful bacteria found only in 12.The organizers of tomorrow's outdoor concert
sewage are difficult to detect directly. Testing for announced that it will go on tomorrow on schedule
E. coli, an easily detected and less harmful type of unless bad weather is forecast or too few advance
bacteria, in ocean water would be a reliable way of tickets are sold. If the concert is canceled, refunds
determining whether or not these more harmful bac- will be made to ticket holders. Since some ticket
teria are present, since ocean water contains E. Coli holders have already been issued refunds even though
only if the water is contaminated with sewage that more than enough advance tickets were sold, it must
contains the harmful bacteria. be the case that bad weather is forecast.

Which of the following, if true, most seriously Which of the following is an error of reasoning con-
weakens the argument? tained in the argument?
(A) There are many different strains of the E. coli (A) It proceeds as if a condition, which by itself is
bacteria, and only some of these strains are enough to guarantee a certain result, is the only
harmful. condition under which that result would occur.
(B) Some types of bacteria found in sewage are (B) It bases a conclusion that is known to require
neither disease-causing nor difficult to detect two conditions on evidence that bears on only
directly. one of those conditions.
(C) Some of the types of bacteria found in sewage (C) It explains one event as being caused by another
along with E. coli are not harmful to people event, even though both events must actually
unless the bacteria are ingested in large have been caused by some third, unidentified
quantities. event.
(D) E. coli dies out much more quickly than some of (D) It treats evidence for the absence of one condi-
the more harmful bacteria found in sewage and tion under which a circumstance would occur
then can no longer be easily detected. as conclusive evidence that that circumstance
(E) Some of the types of bacteria found in sewage will not occur.
along with E. coli reproduce at a slower rate (E) Evidence given to support the conclusion actually
than E. coli. undermines it.

28
13. Although the prevailing supposition has been that it 14. For 20 years all applicants for jobs as technicians at
is too hot for microorganisms to survive deep below the EquipCorp were required to demonstrate that they could
Earth's surface, some scientists argue that there are operate and repair the machinery that was central to
living communities of microorganisms there that have EquipCorp's manufacturing business. Now, however,
been cut off from surface life for millions of years. that particular machinery is obsolete, and very different
These scientists base their argument on the discovery machinery fills the central role. Therefore, the old
of living microorganisms in samples of material that requirement is no longer a useful method for evaluating
were taken from holes drilled as deep as 1.74 miles. whether applicants for jobs as technicians at EquipCorp
have the skills necessary for the job.
The scientists' argument depends on which of the fol-
lowing assumptions? Which of the following, if true, most strengthens the
(A)The microorganisms brought up were of a argument?
species that is related to those previously (A) The machinery that is now obsolete was used by a
known to science. large number of manufacturing companies before
(B)No holes have been drilled into the Earth's it became obsolete.
surface to a distance deeper than 1.74 miles (B) Among the people already holding jobs as tech-
(C)The microorganisms did not come from surface nicians at Equip Corp, those who are most skillful
soil that came into contact with the drilling at operating the new machinery had been some of
equipment. the least skillful at operating the old machinery
(D) The stratum from which the samples came has (C) Most people applying for jobs as technicians today
been below the surface of the Earth ever since have much broader skills than did people applying
the Earth came into existence. for jobs as technicians 20 years ago.
(E) The temperature at the bottom of the holes drilled (D) The skills required to operate and repair the
was not significantly hotter than that of the obsolete machinery are useful in operating and
hottest spots on the Earth's surface. maintaining many other types of machinery at
EquipCorp that are not obsolete.
(E) Much of the machinery that EquipCorp now uses in
manufacturing is very likely to become obsolete
within the next 20 years.

29
练习 5

1. The university's constitution defines the powers of 2.Comparison of the growth rings of ancient trees
two governing bodies.The general council, which enables scientists to determine from a piece of tim-
currently retains authority over constitutional matters, ber the year in which the tree used for the tim-
consists of all the university's living alumni.The ber was felled.Hence, by analyzing the growth rings
twenty-member senate decides routine matters by in timber surviving from ancient buildings, archaeolo-
majority vote.A new proposal suggests making a gists can determine precisely when those buildings
unanimous vote in the senate sufficient to change the were constructed.
constitution.
Which of the following is an assumption on which
If the statements above are true, which of the fol- the argument depends?
lowing must follow on the basis of them? (A) The timber used for construction purposes in
(A) The proposal will take effect only if it receives ancient times was made from very old trees.
the senate's unanimous support. (B) The timber that was used in ancient building
(B) Currently each member of the senate has power construction had not, prior to being used in the
of veto over any proposal that comes before construction of the buildings, lain unused for
that body. an indeterminable amount of time.
(C) Adopting the proposal would allow passage of (C) The growth rings of any tree felled in a given
constitutional changes without the general year are identical to the growth rings of any
council's approval. other tree felled in that year.
(D) Any of the university's alumni can fill any one (D) The oldest of the ancient buildings that survive
of the twenty seats on the senate. to the present day were constructed of the most
(E) Adopting the new proposal would increase the durable woods.
voice of the alumni in the conduct of (E) Ancient builders did not use more than one type
university affairs. of wood for the construction of a given
building.

30
3.Color-tinting of movies originally photographed in 4.In Borkland, university scholarship stipends worth
black-and-white is inappropriate.Hundreds of the $4.2 million were unclaimed last year because several
original artistic choices would have been made scholarship programs attracted no applicants who met
differently had these movies been filmed in color. the programs' qualifying criteria.This is an egregious
Lighting on the sets of these movies was arranged waste of funds in a country where thousands of
to make highlights and shadows look right in promising students each year cannot afford
black-and-white. tuition.Qualifying criteria for those scholarships,
therefore, should be revised.
Which of the following claims, if substituted for the
claim about the lighting of movie sets, would lend Which of the following, if true about Borkland, most
the same type of support to the argument above? strongly supports the conclusion drawn above?
(A) An important part of what gives these movies (A) Many scholarships are unclaimed not because
their identity is the result of the black-and- their qualifying criteria are too restrictive but
white format. because the scholarship programs rarely
(B) Color film would have better captured the film- announce the scholarships' availability.
makers' intentions. (B) Because of inflation, the stipends offered by
(C) Color film is superior to black-and-white film certain scholarship programs now appear less
for making movies. attractive to potential applicants than they did
(D) Makeup for the actors was applied so as to look when they were first offered.
best in black-and-white films. (C) A significant number of scholarship programs
(E) The choice of black-and-white film over color awarded all of their available scholarship
film is entirely a matter of taste. stipends last year, yet most of those programs
have very restrictive qualifying criteria.
(D) Certain scholarship programs accept applications
only from students enrolled in specific major
fields of study, but Borkland's universities no
longer offer courses in the fields specified by
many of those programs.
(E) Several scholarship programs have found it
impossible to revise their qualifying criteria
without engaging in lengthy and costly legal
proceedings.

31
5.In Diersville the new Environmental Action party Which of the following, if true, most seriously
won two seats on the seven-member town council in weakens
1988.It lost both of those seats in the 1992 election, the argument?
even though the party's pro-environment platform had (A) Between 1988 and 1992, the number of eligible
essentially remained unchanged.This decline in the voters in Diersville rose, but not the percentage
party's fortunes clearly demonstrates that in Diersville who actually voted.
environmental concerns faded in significance between (B) Between 1988 and 1992, Diersville's leading
1988 and 1992. political party revised its platform, adopting
a strongly environmentalist stance.
(C) The parties that ran candidates in the 1992 elec-
tion in Diersville were the same as those that
had done so in the 1988 election.
(D) In 1992 the Environmental Action party won
fewer votes in Diersville than it had won in
1988.
(E) Between 1988 and 1992, some measures intended
to benefit the environment had been adopted
by the town council, but with inconclusive
results.

32
6.The United States is not usually thought of as a 7. Until 1984 only aspirin and acetaminophen shared
nation of parakeet lovers.Yet in a census of parakeet the lucrative nonprescription pain-reliever market.
owners in selected comparable countries, the United In 1984, however, ibuprofen was expected to account
States ranked second, with eleven parakeet owners for fifteen percent of all nonprescription pain-reliever
per hundred people.The conclusion can be drawn sales.On that basis business experts predicted for
from this that people in the United States are more 1984 a corresponding fifteen percent decrease in the
likely to own parakeets than are people in most other combined sales of aspirin and acetaminophen.
countries.
Knowledge of which of the following would be most The prediction mentioned in the last sentence above
useful in judging the accuracy of the conclusion? was based on which of the following assumptions?
(A) The number of parakeets in the United States. (A) Most consumers would prefer ibuprofen to both
(B) The number of parakeet owners in the United aspirin and acetaminophen.
States. (B) Aspirin, acetaminophen, and ibuprofen all relieve
(C) The number of parakeet owners per hundred headache pain and muscular aches, but aspirin
people in the country that ranked first in the and ibuprofen can also cause stomach irritation.
census. (C) Before 1984 ibuprofen was available only as a
(D) The number of parakeet owners in the United prescription medicine.
States compared to the numbers of owners of (D) The companies that manufacture and sell aspirin
other pet birds in the United States. and acetaminophen would not also manufacture
(E) The numbers of parakeet owners per hundred and sell ibuprofen.
people in the countries not included in the (E) The introduction of ibuprofen would not increase
census. total sales of nonprescription pain reliever.

33
8. New regulations in Mullentown require manufac- 9. To produce seeds, plants must first produce flowers.
turers there to develop five-year pollution-reduction Two kinds of tarragon plants, Russian tarragon
plans. The regulations require that each manufacturer and French tarragon, look very similar except that
develop a detailed plan for reducing its released pol- Russian tarragon produces flowers and French tar-
lutants by at least 50 percent. Clearly, the regulations ragon does not. The leaves of Russian tarragon,
will not result in significant pollution reduction, how- however, lack the distinctive flavor that makes
ever, since the regulations do not force manufacturers French tarragon a desirable culinary herb
to implement their plans.
If the information presented is true, which of the fol-
Which of the following, if true, most weakens the lowing can most reliably be concluded on the basis
argument? of it?
(A) Mullentown's manufacturing plants are not the (A) As a decorative plant, French tarragon is more
only source of pollution there. desirable than Russian tarragon.
(B) Detailed plans would reveal that measures to (B) The flowers of Russian tarragon plants are prob-
reduce released pollutants would also reduce ably not flavorful.
manufacturers' costs for materials, waste (C) Plants that grow from seeds sold in a packet
disposal, and legal services labeled "tarragon" are not French tarragon.
(C) Pollutants that manufacturing processes create (D) There are no other kinds of tarragon besides
but that are not released directly into the Russian tarragon and French tarragon
environment must nonetheless be collected (E) Garden plants that have flavorful leaves generally
and prepared for disposal do not produce flowers.
(D) Any reductions in pollutants released from
Mullentown's manufacturing plants would
not be noticeable for at least five years.
(E) Each manufacturer will be required to submit its
plan to a committee appointed by Mullentown's
officials.

34
10. In the United States, the financing of industrial 11. Although many brands of gasoline are sold on
research by private industrial firms remained steady Haibei Island, gasoline companies there get all of the
as a percentage of sales during the period between refined gasoline they sell from Haibei seaport's only
1968 and 1978 (after correcting for inflation). But storage tank, which is always refilled with the same
slowdowns in the growth of industrial productivity quality of gasoline. Therefore, the brands of gasoline of
also occurred during that period, a fact that refutes sale on Haibei may be different in name and price, but
the notion that the growth of industrial productivity they are identical in quality.
is directly proportional to the amount invested in
industrial research. The conclusion drawn above depends on which of the
following assumptions?
Which of the following, if true for the United States, (A) Consumers are usually unaware of variations
most weakens the argument above? in the quality of the gasoline they buy unless
(A) Federal funds, which constituted a significant those variations are announced by the gasoline
portion of the support for industrial research companies.
from 1968 to 1978, fell annually and substan- (B) When tankers make gasoline deliveries at
tially during that period. Haibei's seaport, the storage tank on Haibei
(B) The inflation that occurred between 1968 and always receives the same quantity of gasoline
1978 was more severe than leading economists as that in the preceding delivery.
had expected. (C) There is a wide variation in the prices at which
(C) Industrial executives generally favor investing an the different brands of gasoline on Haibei are
appreciably larger portion of corporate funds in sold.
short-term product development than in basic (D) If any gasoline company on Haibei alters the
research. quality of its gasoline before sale, the other
(D) The scientists and engineers who worked in gasoline companies also use methods before
industry from 1968 to 1978 were, as a group, sale that result in the same change in the
more experienced in their jobs than were those quality of their gasoline .
who worked in industry during the previous (E) The gasoline storage tank on Haibei is large
ten-year period. enough to meet the needs of all of Haibei's
(E) Corporate financing of industrial research different gasoline companies.
increased in several of the years immediately
following 1978 (after correcting for inflation)

35
12.A group of paintings made approximately 15,000 Which of the following, if true, best supports the
years ago in a cave in the Loire River valley in what is hypothesis that in painting the animal that resembles a
now France depicts a number of different animals. One chiru the cave artist painted a chiru with which she or he
of the animals depicted seems to resemble the chiru, a was familiar?
rare antelope of the Himalayas. (A) There are numerous representations of imaginary
animals in cave paintings of similar age.
(B) Fossilized remains of a chiru, approximately
16,000 years old, have been found at the northern
end of the valley.
(C) The cave that contains the depiction of an animal
that resembles a chiru contains stylized repre-
sentations of plant life.
(D) Older caves from the same region contain no
representations of animals that resemble a chiru.
(E) The antlers of the animal in the painting are longer
than those of the mature Himalayan chiru.

36
练习 6

1.The ancient Greek playwright Euripides followed the 2.In the United States, average fuel efficiency of
established conventions of verse composition less rig- newly manufactured domestic cars, although
orously at the end of his career than at the beginning. remaining worse than that of newly manufactured
Since the lines from a recently discovered Euripidean imported cars, substantially improved between 1983
play adhere to those conventions as rigorously as do and 1988. Average fuel efficiency of new domestic
lines from Euripides' early plays, the recently discov- cars has not improved since, but the difference in
ered play must have been composed early in Euripides’ average fuel efficiencies of new domestic cars and
career. new imported cars has steadily decreased.

Which of the following is an assumption made in the If the statements above are true, which of the
argument? following must also be true on the basis of them?
(A) All of Euripides' plays were written in verse. (A) Average fuel efficiency of domestic cars manu-
(B) Euripides did not write any plays late in his factured after 1988 was better than that of
career in which he imitated the style of his imported cars manufactured before 1988.
early plays. (B) Average fuel efficiency of newly manufactured
(C) Euripides grew increasingly unaware of the domestic cars has steadily worsened since 1988.
established conventions of verse composition (C) Average fuel efficiency of newly manufactured
as his career progressed. imported cars has steadily worsened since 1988.
(D) Late in his career, Euripides was the only (D) Average fuel efficiency of newly manufactured
playwright of his day who consciously broke imported cars has steadily improved since 1983.
with the established conventions of verse (E) Average fuel efficiency of imported cars manu-
composition. factured in 1983 was better than that of
(E) Ancient playwrights tended to be less willing to imported cars manufactured in 1988.
violate certain conventions early in their
careers than they were late

37
3. Between 1970 and 1980, energy consumption by 4.Many people acquire software programs for their
United States industry peaked and then declined, so home computers by illegally copying those programs
that by 1980 total industrial use of energy was below rather than purchasing them. People who own home
the 1970 level even though total industrial output computers must be making, on average, fewer illegal
had grown substantially in the same period. Industry copies of software programs than before, however,
must have instituted highly effective energy conser- since the average number of software programs that
vation measures in those years to have achieved such people purchase to use on their home computers has
impressive results. increased substantially over the past five years.

Which of the following, if true, most seriously weak- Which of the following, if true, most strengthens the
ens the conclusion of the argument? argument?

(A) Many industries switched to the greatest extent (A) The number of home computers in use has
possible from high-priced oil to lower-priced increased substantially over the past five years.
alternatives throughout the 1970's. (B) Five years ago, about half of the software pro-
(B) Total residential energy consumption was higher grams used on home computers had been
in the United States in 1980 than it had been illegally copied rather than purchased.
in 1970 (C) Most people who have home computers use
(C) Many industrial users of energy had paid little their computers more frequently the longer
attention to energy conservation prior to 1970. they have them.
(D) Industrial output grew less rapidly from 1970 to (D) Few people who prefer to copy computer
1980 than it had from 1960 to 1970. software programs illegally cannot copy the
(E) The industries whose production dropped sharply software programs they want because they
during the 1970's included a disproportionately have no acquaintances who have those
large number of energy-intensive industries. software programs.
(E) On average, people with home computers have
the same number of software programs today as
people with home computers did five years ago.

38
5.From a newspaper editorial: 6.Stem borers are insect pests that often ruin North
Many people who are addicted to heroin will even- American corn crops. On some other continents, crop
tually attempt to overcome their addiction, prin- damage by stem borers is controlled by a certain
cipally for two reasons:the expense of maintaining species of wasp. Since these wasps eat nothing but
a heroin addiction and the fear of arrest. If heroin stem borers, importing them into North America will
were legalized and made available cheaply, as some keep crop damage from stem borers under control
people advocate, neither of these reasons would without endangering other North American insect
apply species.

The considerations above can best serve as part of an Which of the following is an assumption on which the
argument that argument depends?
(A) legalizing the sale of heroin would cause the (A) Corn is the principal food of stem borers that
price of this drug to go down live on continents other than North America.
(B) making it easier for heroin addicts to obtain (B) The wasps are capable of surviving in North
treatment for their addiction would encourage America long enough to eat significant
many heroin addicts to attempt to overcome numbers of stem borers.
their addiction (C) No wasp in North America is closely related to
(C) legalizing the sale of heroin would increase the the species of wasp that eats stem borers.
number of crimes committed by heroin addicts (D) On continent other than North America, the
to support their addiction wasps control stem borers more effectively
(D) making heroin available legally and cheaply than does any other pest control measure.
would make it less likely that heroin addicts (E) Corn crops on continents other than North
will attempt to overcome their addiction America are not damaged by any insect pests
(E) decreasing the severity of penalties for individuals other than stem borers.
who use heroin would not increase the number
of new heroin addicts

39
7.In the country of Laurelia, legal restrictions on the sale Which of the following, if true, gives the strongest
of lock-picking equipment were relaxed ten years ago, support to the argument?
and since then Laurelia's burglary rate has risen (A) Laurelia's overall crime rate has risen
dramatically. Hence, since legally purchased dramatically over the last ten years.
lock-picking equipment was used in most burglaries, (B) There is wide popular support in Laurelia for the
reintroducing strict limits on the sale of this equipment reintroduction of strict limits on the sale of lock-
would help to reduce Laurelia's burglary rate. picking equipment.
(C) The reintroduction of strict limits on the sale of
lock-picking equipment in Laurelia would not
prevent legitimate use of this equipment by
police and other public safety officials.
(D) Most lock-picking equipment used in Laurelia is
fragile and usually breaks irreparably within a
few years of purchase.
(E) The introduction five years ago of harsher punish-
ments for people convicted of burglary had little
effect on Laurelia's burglary rate.

40
8.Salesperson:The picture tube is the central com- Which of the following is an assumption that, if
ponent of any television, and Picturesque tele- justified, would allow the conclusion of the sales-
visions use the same high-quality picture tubes person's argument to be properly drawn?
as those used in TruVue televisions. Since you
pay a much lower price for a Picturesque, you (A) TruVue televisions are much more widely
pay a lower price for a television with the same advertised than are Picturesque televisions.
picture quality when you buy a Picturesque (B) The picture quality of a television is determined
instead of a TruVue. solely by the quality of its picture tube.
(C) A salesperson earns much less on the sale of
each Picturesque television than on the sale of
each TruVue television.
(D) Many more Picturesque televisions are sold
each year than TruVue television.
(E) Picturesque televisions are assembled in the
same factory that assembles TruVue tele-
visions.

41
9.In a recent film set in seventeenth-century Europe, the 10.The government of Pontran claims that Tor City, one
hero is seen doing the crawl, a swimming stroke not of the six major cities in that country, is alone among
known in Europe before the 1920's.However, since Pontran's cities in having sustained strong job growth
moviegoers obviously are not experts in the history of this year. Clearly, however, any job growth this year.
swimming strokes, for most of the film's audiences Clearly, however, any job growth there must be purely
this imaginary; in fact, in Tor City and only there, more
blunder clearly cannot have interfered with whatever people are unemployed this year than were last year.
sense of historical authenticity the film otherwise
achieved. The argument countering the government's claim
depends on the assumption that:
Which of the following, if true, most seriously (A) unemployed workers in Pontran did not, in
weakens the argument given? significant numbers, move to Tor City when the
(A) The film was widely praised for being historically government claimed Tor City had strong job
plausible, even though it portrayed many events growth
that were not historically attested. (B) the unemployment rate in Tor City is higher year
(B) The scene that shows the film's hero doing the than in any previous year
crawl is a rescue scene pivotal to the film's (C) actions taken by the government of Pontran
action, significantly affect the unemployment rate in Tor
and parts of it are even shown a second time, in a City
flashback. (D) the unemployment rate in Tor City, though
(C) Makers of historical films, even of those set as increased, is still the lowest of any city in Pontran
recently as the nineteenth century, routinely strike (E) there is no significant seasonal variation in the
compromises between historical authenticity and unemployment rate for Pontran as a whole
the need to keep their material accessible to a
modern audience, as in the actors' speech
patterns.
(D) The crawl that European swimmers used in the
1920's was much less efficient and more
awkward-looking than the crawl that is currently
taught.
(E) A slightly earlier film featuring an eighteenth
century sea battle in Europe was ridiculed in
numerous popular reviews for the historical lapse
of showing a sailor doing the crawl in swimming
to safety.

42
11.As people age, the number of calories they need each Which of the following would be most helpful to
day decreases while their daily requirement of vitamin know to evaluate the argument?
B6 increases. Clearly, unless older people take B 6 (A) Whether the relative decrease in the daily
supplements or eat a diet that contains more B 6 than did requirement of calories is greater than the relative
the diet they ate when they were young adults, there is increase in the daily requirement of vitamin B 6.
little likelihood that they will get B 6 in needed amounts. (B) Whether the form of vitamin B6 found in dietary
supplements is more readily absorbed and utilized by
the body than is the form of this vitamin that
is found in food.
(C) Whether the consequences of not getting vitamin
B6 in required daily amounts are more serious for
older people than for young adults.
(D) Whether the diets of most people, when they are
young adults, include vitamin B6 in amounts far
in excess of their daily needs.
(E) Whether the diets of older people are more likely
than those of young adults to include one or more
meals a day that are devoid of foods containing
vitamin B6.

43
练习 7

1. The Environmental Protection Agency must 2. Aedes albopictus, a variety of mosquito that has
respond to the hazard to children's health recently established itself in the southeastern
posed by exposure to asbestos fibers released United States, is less widespread than the
in the air in school classrooms. Since it is indigenous swamp mosquito. Both the swamp
impossible to close school buildings, the best plan mosquito and A. albopictus can carry viruses that
would be to initiate programs that mandate the are sometimes fatal to humans, but A. albopictus
immediate removal of asbestos from all the is a greater danger to public health.
school buildings that are found to contain
Each of the following, if true, provides additional
asbestos, regardless of whether or not the
information that strengthens the judgment given
buildings are in use.
about the danger to public health EXCEPT:
Which of the following, if true, is the strongest
(A) Unlike the swamp mosquito, A. albopictus
reason for the Environmental Protection Agency
originated in Asia, and larvae of it were not
not to follow the plan outlined above?
observed in the United States before the mid-
(A) The techniques available for removing
1980's.
asbestos often increase the level of airborne
(B) Unlike the swamp mosquito, A. albopictus
asbestos.
tends to spend most of its adult life near
(B) Schools are places where asbestos is
human habitation.
especially likely to be released into the air by
(C) Unlike swamp mosquito larvae, A.
the action of the occupants.
albopictus larvae survive in flower pots, tin
(C) Children exposed to airborne asbestos run a
cans, and many small household objects that
greater risk of developing cancer than do
hold a little water.
adults exposed to airborne asbestos.
(D) In comparison with the swamp mosquito, A.
(D) The cost of removing asbestos varies from
albopictus hosts a much wider variety of
school to school, depending on accessibility
viruses known to cause serious diseases in
and the quantity of asbestos to be removed.
humans.
(E) It is impossible to determine with any degree
(E) A. albopictus seeks out a much wider range
of certainty if and when construction materials
of animal hosts than does the swamp mosq-
that contain asbestos will break down and
uito, and it is more likely to bite humans.
release asbestos fibers into the air.

44
3. A person's cholesterol level will decline 4. A certain type of dinnerware made in Ganandia
significantly if that person increases the contains lead. Lead can leach into acidic foods,
number of meals eaten per day, but only if and Ganandians tend to eat highly acidic foods.
there is no significant increase in the amount However, the extreme rarity of lead poisoning in
of food eaten. However, most people who Ganandia indicates that the dinnerware does not
increase the number of meals they eat each day contain dangerous amounts of lead.
will eat a lot more food as well.
Which of the following, if true, most seriously
weakens the argument above?
If the statements above are true, which of the
(A) The dinnerware is produced exclusively for
following is most strongly supported by them?
sale outside Ganandia.
(A) For most people, cholesterol level is not
(B) Ganandian foods typically are much more
significantly affected by the amount of food
acidic than foods anywhere else in the
eaten per day.
world.
(B) For most people, the amount of food eaten per
(C) The only source of lead poisoning in
meal is most strongly affected by the time of
Ganandia is lead that has leached into food.
day at which the meal is eaten.
(D) Most people who use the dinnerware are not
(C) For most people, increasing the number of
aware that it contains lead.
meals eaten per day will not result in a
(E) Acidic foods can leach lead from dinnerware
significantly lower cholesterol level.
even if that dinnerware has a protective
(D) For most people, the total amount of food
coating.
eaten per day is unaffected by the number of
meals eaten per day.
(E) For most people, increasing the number of
meals eaten per day will result in a significant
change in the types of food eaten.

45
5. Sergeant 6. Because adult iguanas on Plazos Island are much
Our police academy no longer requires its smaller than adult iguanas of the same species
applicants to pass a physical examination on nearby islands, researchers assumed that
before being admitted to the academy. As a environmental conditions on Plazos favor the
result, several candidates with weak hearts and survival of relatively smaller baby iguanas
high blood pressure have been admitted. Hence, (hatchlings) in each yearly brood. They
we can expect our future police force to have discovered instead that for each of the past three
more health problems than our current police years, 10 percent of the smaller and 40 percent
force. of the larger hatchlings survived, because larger
hatchlings successfully evade their predators.
Knowledge of each of the following would be
relevant to determining the reliability of the Which of the following, if true about Plazos but
sergeant's prediction EXCEPT whether not about nearby islands, contributes most to an
(A) police officer candidates are screened for explanation of the long-standing tendency of
high blood pressure before joining the police iguanas on Plazos to be smaller than those of the
force same age on nearby islands?
(B) the police officer candidates who are not (A) Periodic wind shifts cause extended dry
healthy now are likely to be unhealthy as spells on Plazos every year, putting the larger
police officers iguanas, whose bodies require relatively
(C) graduates of the police academy are required more water, at a great disadvantage.
to pass a physical examination (B) There are exactly three species of iguanas on
(D) the health of the current police officer Plazos but only two species of seagulls that
candidates is worse than was the health of feed on iguanas, and a relatively small
police officer candidates in the past percentage of each year's hatchlings are
(E) a police officer's health is a reliable indicator consumed by seagulls.
of the officer's performance (C) Wild cats, which were introduced as pets by
early settlers and which were formerly major
predators of Plazos iguanas, were recently
killed off by a disease specific to cats.
(D) The iguanas on Plazos are a relatively
ancient part of the island's animal life.
(E) Both land and marine iguanas live on Plazos,
and the land iguanas tend to be larger than
marine iguanas of the same age.

46
7. Every human being who has ever lived had two 8. Each of the academic journals Thought and Ergo
parents. Therefore, more people were alive three has a review committee to prevent misattributed
thousand years ago than are alive now. quotations from appearing in its published
articles. Nevertheless, about ten percent of the
The reasoning in the argument is flawed because quotations in Thought's published articles are
it misattributed, whereas Ergo contains no
(A) overlooks the number of people in each misattributions. Ergo's committee is more
generation during the last three thousand effective, therefore, than Thought's at finding
years who left no descendants misattributed quotations.
(B) disregards possible effects of disasters such The argument above assumes that
as famines and plagues on human history (A) most of the articles submitted to Thought for
(C) overestimates the mathematical effect of publication contain misattributed quotations
repeated doublings on population size (B) there are at least some misattributed
(D) fails to take into account that people now quotations in articles submitted to Ergo for
alive have overlapping sets of ancestors publication
(E) fails to consider that accurate estimation of (C) the members of Ergo's committee are, on the
the number of people alive three thousand whole, more knowledgeable than are the
years ago might be impossible members of Thought's committee
(D) the number of misattributed quotations in a
journal is an accurate measure of how
carefully that journal is edited
(E) the authors who submit articles to Ergo for
publication are more thorough in attributing
quotations than are the authors who submit
articles to Thought

47
9. In the last few decades, grassy wetlands, essential 10. A researcher found that, in proportion to their
to the nesting and breeding of ducks, geese, body weights, children eat more carbohydrates
swans, and most other species of waterfowl, have than adults do. Children also exercise more than
been extensively drained and cultivated in adults do. The researcher hypothesized that
southern Canada and the northern United States, carbohydrate consumption varies in direct
Duck populations in North American have proportion to the calorie demands associated with
plummeted during this time, but populations of different levels of exercise.
swans and geese have been affected less
Which of the following, if true, most seriously
dramatically.
undermines the researcher's hypothesis?
(A) More carbohydrates are eaten per capita in
Which of the following, if true, most helps to
nations where the government spends more
explain the difference mentioned above?
per capita on public exercise programs.
(A) Prohibition of hunting of waterfowl is easier
(B) Children who do not participate in organized
to enforce in areas under cultivation than in
sports tend to eat fewer carbohydrates than
wild lands.
children who participate in organized sports.
(B) Most geese and swans nest and breed farther
(C) Consumption of increased amounts of
north than ducks do, in areas that still are not
carbohydrates is a popular tactic of runners
cultivated.
preparing for long-distance races.
(C) Land that has been harvested rarely provides
(D) Periods of physical growth require a
food suitable for waterfowl.
relatively higher level of carbohydrate
(D) Goose and swan populations decline in
consumption than otherwise.
periods of drought, when breeding sites are
(E) Though carbohydrates are necessary for the
fewer.
maintenance of good health, people who
(E) Because they are larger than ducks, geese and
consume more carbohydrates are not
swans have a harder time finding protected
necessarily healthier.
nesting sites in areas that are cultivated.

48
11. Experts removed a layer of eighteenth-century 12. Although it is assumed that peacocks'
red paint from a figure in a painting by a magnificent tails function essentially to attract
sixteenth-century Italian artist, revealing a layer peahens, no one knows why it should be
of green paint underneath. Since the green paint magnificent tails that give a competitive
dates from the sixteenth century, the figure must advantage in securing mates. One explanation is
have been green, not red, when the painting was that peahens are more likely to mate with
completed in 1563.Which of the following, if peacocks with magnificent tails than with
true, most seriously weakens the argument? peacocks that lack magnificent tails.
(A) The experts had been commissioned to
restore the painting to the colors it had when Which of the following is an error of reasoning
it was completed. exemplified by the explanation?
(B) X-rays reveal an additional layer of paint (A) Attributing to animals qualities that are
beneath the green paint on the figure. characteristically human
(C) Chemical analyses were used to determine (B) Extending a conclusion that is true of only
the ages of the red paint and the green one species of a genus to all species of the
paint. genus
(D) The red paint was added in the eighteenth (C) Offering as an explanation a hypothesis that
century in an attempt to repair damage done in principle can be neither verified nor
in the late seventeenth century. proved false
(E) Red paint on the robe of another figure in the (D) Offering the phenomenon that is to be
painting dates from the sixteenth century. explained as the explanation of that
phenomenon
(E) Assuming without warrant that peacocks
with magnificent tails are likely to have
other features strongly attractive to peahens

49
13. Whenever a French novel is translated into The recommendation is based on which of the
English, the edition sold in Britain should be in following assumptions?
British English. If the edition sold in Britain (A) The authors of French novels are usually
were in American English, its idioms and native speakers of French.
spellings would appear to British readers to be (B) A non-British reader of a novel written in
strikingly American and thus to conflict with the British English will inevitably fail to
novel's setting. understand the meanings of some of the
words and idioms in the novel.
(C) No French novel that is to be sold in Britain
in English translation is set in the United
States.
(D) A British reader of a British novel will
notice that the idioms and spellings used in
the novel are British.
(E) Most French novels are not translated into
both British English and American English.

50
练习 8

1. Auditor from Acme Industries: Last week at 2. Spiders of many species change color to match
Acme Bakery, about six percent of the pastries the pigmentation of the flowers they sit on. The
baked during the night shift were found to be insects preyed on by those spiders, unlike human
imperfect, but no imperfect pastries were found beings, possess color discrimination so acute that
among those baked during the day shift. Pastries they can readily see the spiders despite the
are inspected during the same shift in which they seeming camouflage. Clearly, then, it must be in
are baked, so clearly the night-shift quality control evading their own predators that the spiders’
inspectors were more alert, despite their nighttime color changes are useful to them.
work hours, than the dayshift quality control
inspectors. Which of the following, if true, most strengthens
the argument?
The argument depends on the assumption that (A) Among the animals that feed on color-
(A) at least some imperfect pastries were baked changing spiders are a few species of bat,
during the day shift at Acme Bakery last which find their prey through sound echoes.
week (B) Certain animals that feed on color-changing
(B) not all of the pastries that the night-shift spiders do so only sparingly in order to keep
quality control inspectors judged to be from ingesting harmful amounts of spider
imperfect were in fact imperfect venom.
(C) the night-shift quality control inspectors (C) Color-changing spiders possess color
received more training in quality control discrimination that is more acute than that of
procedures than did the day-shift quality spiders that lack the ability to change color.
control inspectors (D) Color-changing spiders spin webs that are
(D) in a normal week, fewer than six percent of readily seen by the predators of those spiders.
the pastries baked during the night shift at (E) The color discrimination of certain birds that
Acme Bakery are found to be imperfect feed on color-changing spiders is no more
(E) there are only two shifts per day at Acme acute than that of human beings.
Bakery, a day shift and a night shift

51
3. Which of the following most logically completes 4. V-shaped walled structures in central Asia
the argument below? were used by prehistoric hunters who drove
hoofed animals into an enclosure at the point
Each year every employee of SAI Corporation of the V. The central Asians who built these
must enroll in one of the two health insurance structures probably learned this hunting
plans offered by SAI. One plan requires a technique from invaders from southwest Asia,
sizable monetary contribution from employees; because the arrival of invaders from a region
the other plan is paid for entirely by SAI. Many in southwest Asia where similar structures had
SAI employees enroll in the plan requiring long been used coincides roughly with the
employee contributions. This fact does not show building of the earliest of such structures in
that they feel that this plan’s benefits are central Asia.
superior to those provided by the plan requiring
no employee contribution since----. Which of the following, if true, most strengthens
(A) the plan that requires an employee contribution the argument?
costs and enrolled employee significantly less (A) Excavations in the central Asian region do
per year than do typical health insurance not indicate whether invaders from
plans offered by corporations other than SAI southwest Asia settled permanently in
(B) only SAI employees who have worked for central Asia.
SAI for at least fifteen years are eligible to (B) The V-shaped structures in central Asia
enroll in the plan paid for entirely by SAI were roughly 70 meters long, whereas the
(C) the two health insurance plans currently similar structures in southwest Asia were
offered by SAI are substantially the same usually over 300 meters long.
plans SAI has offered for the past ten years (C) The walls of the structures in central Asia
(D) most of the SAI employees enrolled in the were made from earth, whereas the walls
plan paid for entirely by SAI are under 50 of the structures in southwest Asia
years old were made of rock.
(E) both plans offered by SAI provide benefits (D) The earliest examples of V-shaped walled
not only for employees of SAI but also for structures in central Asia were of an
children and spouses of enrolled employees advanced design.
(E) Some of the walled structures used for
hunting in southwest Asia were built well
after the earliest such structures were built
in central Asia.

52
5. Which of the following most logically 6. When amphibians first appeared on Earth millions
completes the argument? of years ago, the amount of ultraviolet radiation
penetrating Earth’s atmosphere was much greater
Virtually all respondents to a recent voter than it is today. Therefore, current dramatic
survey reported allegiance to one of the two decreases in amphibian populations cannot be the
major political parties. But over a third of the result of recent increases in ultraviolet radiation
voters from each party reported being so penetrating Earth’s atmosphere.
disenchanted with the governing philosophies
of both parties that they might join a third Which of the following is an assumption on which
major party if one were formed. Even if this the argument depends?
poll reflects general voter sentiment, however, (A) The eggs of modern amphibians are not
there is no chance that a new party could significantly more vulnerable to ultraviolet
attract a third of all voters, since----. radiation than the eggs of the first
(A) the current level of disenchantment with the amphibians were.
governing philosophies of the two major (B) Modern amphibians are not as likely as the
parties is unprecedented first amphibians were to live in habitats that
(B) the disenchanted members of the two major shield them from ultraviolet radiation.
parties are attracted to very different (C) Populations of modern amphibians are not
governing philosophies able to adapt to changing levels of radiation
(C) most respondents overestimated the proportion as readily as populations of early amphibians
of voters disenchanted with both parties, saving were.
that the proportion was more than 50 percent (D) The skin of amphibians is generally more
(D) nearly half of all respondents reported that sensitive to ultraviolet radiation than the
they would be more likely to cease voting skin of other animals is.
altogether than to switch their party affiliation (E) The skin of amphibians is less sensitive to
(E) any new party would be likely to inspire ultraviolet radiation than to other types of
citizens who have not voted before to join radiation.
and to become regular voters

53
7. Chris: Hundreds of traffic accidents annually are 8. According to ancient records, the first tax that the
attributable to the poor condition of our city’s government of Selea imposed on a basic commodity
streets. The streets must therefore be repaired to was a tax of two centima coins on every jar of cooking
save lives. oil sold in Selea. Tax records show that despite a stable
population and strict enforcement of tax laws, revenues
Leslie: For less than the cost of those repairs, the city from the oil tax declined steeply over the first two years
could improve its mass transit system and thus that the tax was in effect.
dramatically reduce traffic congestion, which
contributes significantly to those traffic accidents. Which of the following, if true, most helps to
The city cannot afford to do both, so it should explain the decline in Selean oil-tax revenues?
improve mass transit, because reduced traffic
congestion has additional advantages. (A) During the decade following the
implementation of the tax, the average
Which of the following best describes the point household income in Selea rose steadily.
at issue between Chris and Leslie? (B) Two years after implementing the tax on
(A) Whether a certain problem in fact exists cooking oil, the Selean government began
(B) How a certain problem came into being to implement taxes on numerous other
(C) Who is responsible for addressing a certain basic commodities.
problem (C) Jars of cooking oil were traditionally
(D) Whether the city has sufficient financial bought as wedding gifts in Selea at the time
resources to address a certain problem the tax went into effect, and gifts of
(E) How the city can best address a certain cooking oil increased after the
problem implementation of the tax.
(D) After the tax was imposed., Selean
merchants began selling cooking oil in
larger jars than before.
(E) Few Selean households began to produce
their own cooking oil after the tax was
imposed.

54
9. Housing construction materials give off distinctive 10. In December 1992 Tideville Shopping Mall repaired
sounds when exposed to high temperatures. Acoustic and improved the lighting in the mall’s parking lots,
sensors accurately detect such sounds and fire alarms and in 1993 car thefts and attempted car thefts from
incorporating acoustic sensors can provide an early those lots decreased by 76 percent from the previous
warning of house fires, allowing inhabitants to escape year. Since potential car thieves are generally
before being overcome by smoke. Since smoke deterred by good lighting, the decrease can be
inhalation is the most common cause of fatalities in attributed to these improvements.
house fires, mandating acoustic-sensor-based alarms
instead of smoke detectors will eliminate house fire as Which of the following, if true, most helps to
a major cause of death. strengthen the argument above?
(A) Both in 1992 and in 1993, most of the cars
Which of the following, if true, most weakens
stolen from the mall’s parking lots were
the argument given?
relatively new and expensive luxury
(A) The present high cost of acoustic-sensor-
models.
based alarm systems will decline if their
(B) Most of the cars that were stolen from the
use becomes widespread.
mall in 1992 were stolen between 11 A. M.
(B) When fully ignited, many materials used in
and 4 P.M.
housing construction give off sounds that are
(C) Tideville Shopping Mall is one of only
audible even from several hundred yards away.
three shopping malls in the Tideville area.
(C) Many fires begin in cushions or in
(D) In the town of Tideville, where the mall is
mattresses, producing large amounts of
located, the number of car thefts was about
smoke without giving off any sounds.
the same in 1993 as in 1992.
(D) Two or more acoustic-sensor-based alarms
(E) In 1993 the number of security officers
would be needed to provide adequate
patrolling the mall’s parking lots at night
protection in some larger houses.
was doubled.
(E) Smoke detectors have been responsible for
saving many lives since their use became
widespread.

55
11. Legislator: We should not waste any more of the 12. Which of the following most logically
taxpayers’ money on the government’s job-creation completes the argument?
program. The unemployment rate in this country has Each year a consumer agency ranks all domestic
actually risen since the program was begun, so the airlines for on-time performance during the previous
program has clearly been a failure. year, using as its sole criterion the percentage of
each airline’s flights that left no more than fifteen
Which of the following is an assumption on which minutes late. The agency does not count delays due
the legislator’s argument depends? to mechanical reasons, but the fact that the
(A) The budget of the job-creation program has percentage of delayed flights hat were delayed for
typically increased every year. mechanical reasons was approximately the same for
(B) The unemployment rate would not have all domestic airlines last year means that ----- .
risen even more than it has if the job-
creation program had not been in existence. (A) including delays for mechanical reasons in
(C) The unemployment rate is higher now than calculating the airline rankings for on-
at any time before the inception of the time performance would have had little, if
job-creation program. any, effect on last year’s rankings
(D) If the job-creation program had been run (B) airlines would work harder to reduce
more efficiently, it could have better delays if delays for mechanical reasons
served its purpose. were included in the determination of on-
(E) Other government programs are no more time performance rankings
effective in reducing unemployment than is (C) the agency’s rankings do not give
the job-creation program. consumers an accurate idea of how a given
airline compares to other airlines with
respect to the percentage of flights delayed
last year
(D) those airlines with the best on-time
performance record last year also had the
greatest number of delays for mechanical
reasons
(E) on-time performance was approximately
the same for all domestic airlines last year

56
14. Pollutants in the atmosphere can cause acid rain
13. No one can be licensed as an electrician in Parker (rain with high acidity levels). While acid rain in
County without first completing a certain course in itself cannot significantly affect the acidity of bodies
electrical safety procedures. All students majoring in of water into which it falls, it can greatly increase
computer technology at Parker County Technical the acidity of nearby lakes by increasing the amount
College must complete that course before graduating. of decaying matter on a forest floor. A recent
Therefore, any of the college’s graduates in increase in the acidity of the water in Forest Lake,
computer technology can be licensed as an therefore, surely indicates that the rain falling nearby
electrician in Parker County. has become more acid.

The answer to which of the following would be


Which of the following, if true, most seriously
most helpful in evaluating the argument?
weakens the argument?
(A) Is a college degree a requirement for being
(A) Even in areas without significant amounts
licensed as an electrician in Parker County?
of acid rain, most lakes in regions with
(B) Do all students majoring in computer
vegetation similar to the vegetation around
technology who complete the course in
Forest Lake have acidity levels higher than
electrical safety procedures at Parker
those of other lakes.
County Technical College eventually graduate?
(B) Recent air-quality tests in the region around
(C) Is completion of a course in electrical
Forest Lake have revealed a slight increase in
safety procedures the only way a person
the amount of pollutants in the air.
licensed as an electrician in Parker County
(C) Large-scale logging, which was recently
can have learned those procedures?
begun in the forest surrounding Forest
(D) Is a period of practical apprenticeship a
Lake, has increased the amount of decaying
requirement for becoming a licensed
matter on the forest floor.
electrician in Parker County but not for
(D) There is some disagreement among scientists
graduating from the college in computer
about exactly how pollutants in the atmosphere
technology?
cause acid rain.
(E) Do any of the students at Parker County
(E) Decaying matter exists on all forest floors
Technical College who are not majoring in
and is an important factor in maintaining
computer technology take the course in
the healthy growth of the forests.
electrical safety procedures?

57
15. Most of Earth’s surface is ocean. The ocean floor is Which of the following, if true, provides the
inaccessible for extensive research without most support for the conclusion?
equipment of greater technological sophistication (A) Many mountain ranges lie entirely beneath the
than is currently available. It must therefore be true ocean surface, yet new underwater surveying
that scientists know less about the ocean floor equipment has produced three-dimensional charts
environment than about almost any other of them that are as accurate as those available for
environment on Earth. mountain ranges on land.
(B) Strong water currents circulate on the ocean
floor, but the general pattern of their
movement is not so well understood as is the
pattern of air currents that circulate over
land.
(C) In contrast to most land environments,
temperature conditions at the ocean floor are
generally stable and uniform, since sunlight
does not penetrate far below the ocean
surface.
(D) Very few people have seen detailed maps
of extended regions of the ocean floor,
even though such maps are available in
almost all large libraries.
(E) Animals living on the ocean floor must be
able to withstand water pressure that is far
greater than the atmospheric pressure with
which land animals live.

58
练习 9

(在阅读套题训练之前勿做。

2. Which of the following most logically completes


1. At the Shadybrook dog kennel, all the adult animals the argument?
were given a new medication designed to reduce a Alivia’s government has approved funds for an
dog’s risk of contracting a certain common infection. electricity-generation project based on the
Several days after the medication was administered, construction of a pipeline that will carry water
most of the puppies of these dogs had elevated from Lake Cylus, in the mountains, to the much
temperatures. Since raised body temperature is a side smaller Lake Tifele, in a nearby valley. The
effect of this medication, the kennel owner amount of electricity generated will be
hypothesized that the puppies’ elevated temperatures insufficient by itself to justify the project’s cost,
resulted from the medication’s being passed to them even if the price of imported oil-Alivia’s primary
through their mothers’ milk. source of electricity-increases sharply.
Nonetheless, the pipeline project is worth its cost,
Which of the following, if true, provides the most
because ——
support for the kennel owner’s hypothesis?

(A) Some puppies have been given the new (A) the price of oil, once subject to frequent sharp
medication directly but have not suffered elevated increases, has fallen significantly and is now
temperatures as a side effect. fairly stable
(B) The new medication has been well received by (B) the project could restore Lake Tifele, which is
dog breeders as a safe and effective way of currently at risk of drying up and thus of being
preventing the spread of certain common canine lost as a source of recreation income for Alivia
infections. (C) the government of Alivia is currently on excellent
(C) None of the four puppies in the kennel who had terms with the governments of most of the
been bottle-fed with formula had elevated countries from which it purchases oil
temperatures. (D) it would cost less to generate electricity by
(D) an elevated temperature is a side effect of a moving water from Lake Cylus to lake Tifele
number of medications for dogs other than the than to do so by moving water from Lake Cylus
new medication administered at the kennel. to another valley lake
(E) Elevated temperatures such as those suffered by (E) Alivian officials do not expect that the amount of
most of the puppies in the kennel rarely have electricity used in Alivia will increase
serious long-term effects on a puppy’s health. substantially within the next ten years

59
3. Amusement rides at permanent fairgrounds are 4. When cut, the synthetic material fiberglass, like
dismantled once a year for safety inspections by asbestos, releases microscopic fibers into the air. It is
independent consultants. Traveling fairs, which known that people who inhale asbestos, fibers suffer
relocate each month, can slip past the net of safety impairment of lung functions. A study of 300 factory
inspections and escape independent inspection for workers who regularly cut fiberglass showed that their
several years. Therefore, the rides at traveling fairs lung capacity is, on average, only 90 percent of that of a
are less safe than the rides at permanent fairs. comparable group of people who do not cut fiberglass.

The statements above, if true, most strongly support


Which of the following, if true about traveling fairs, which of the following hypotheses?
most seriously weakens the argument? (A) People who work with fiberglass are likely also
(A) Before each relocation, the operators dismantle to work with asbestos.
their rides, observing and repairing potential (B) Fiberglass fibers impair lung function in people
sources of danger, such as worn ball bearings. who inhale them.
(B) their managers have less capital to spend on the (C) Fiberglass releases as many fibers into the air
safety and upkeep of the rides than do managers when cut as does asbestos.
of permanent fairs. (D) Coarse fibers do not impair lung function in
(C) Since they can travel to new customers, they people who inhale them.
rely less on keeping up a good reputation for (E) If uncut, fiberglass poses no health risk to
safety. people who work with it.
(D) While they are traveling, the fairs do not receive
notices of equipment recalls sent out by the
manufacturers of their rides.
(E) The operators of the rides often do not pay
careful attention to the instructions for operating
their rides.

60
6.Like most other coastal towns in Norway, the town of
5. Politician: Pundits claim that by voting for candidates Stavanger was quiet and peaceful until the early
who promise to cut taxes, people show that they 1960's, when it became Norway's center for offshore
want the government to provide fewer services than oil exploration. Between then and now, violent
it has been providing. By that reasoning, however, crime and vandalism in Stavanger have greatly
people who drink too much alcohol at a party in the increased. Stavanager's social problems probably
evening want a headache the next morning. resulted from the oil boom, since violent crime and
Which of the following could replace the statement vandalism have remained low in coastal towns in
about people who drink too much without Norway that have had no oil boom.
undermining the force of the politician's argument?
Which of the following most accurately describes the
(A) People who spend more money than they can
method of reasoning employed in the argument?
afford want the things they spend that money
on. (A) Arguing that a circumstance is not a
(B) People who seek different jobs than they precondition for a phenomenon on the grounds
currently have do not want to work at all. that the phenomenon sometimes occurs where
(C) People who buy new cars want to own cars that the circumstance is not present
are under manufacturer's warranty. (B) Arguing that a circumstance is a cause of a
(D) People who decide to stay in bed a few extra phenomenon on the grounds that the
minutes on a workday morning want to have to phenomenon has not occurred where the
rush to arrive at work on time. circumstance is not present
(E) People who buy lottery tickets want the (C) Arguing that a particular thing cannot have
economic freedom that winning the lottery caused a phenomenon because that thing was
would bring. not present before the phenomenon occurred
(D) Attempting to establish a claim by arguing that
the denial of the claim is inconsistent with the
observed facts
(E) Attempting to establish that certain
circumstances that would have had to occur for
a particular explanation to be correct could not
have occurred

61
7. Excavations at a Mayan site have uncovered jewelry 8. Over the last 40 years there has been a great increase
workshops located some distance from the center of not only in the number of agricultural pesticides in
the site on roads radiating outward from the center. use but also in the care and sophistication with
Since the nobility lived only in the area of the center, which they are used by farmers. Nevertheless, the
archaeologists conclude that these workshops made proportion of agricultural crops lost to certain pests
jewelry, not for the nobility, but for a middle class worldwide has increased over the same period, even
that must have been prosperous enough to afford it. when the pests concerned have not developed
resistance to existing pesticides.
The archaeologists' argument assumes which of the
following about the artisans who worked in the
Which of the following, if true. best explains how
workshops'
improvements in pesticide use have been
(A) They were themselves prosperous members of a accompanied by greater losses to certain pests?
middle class.
(A) Some dangerous but relatively ineffective
(B) They lived near their workshops.
pesticides common 40 years ago are no longer
(C) Their products were not made from the same
in widespread use.
materials as was jewelry for the nobility.
(B) As pesticides have become increasingly pest-
(D) They worked full-time at making jewelry and
specific, controlling certain pests with pesticides
did not engage in farming
has turned out to cost more in many cases than
(E) They did not take the jewelry they had made in
the value of crop losses caused by those pests.
the workshops to clients who were members of
(C) Because today's pesticides typically have more
the nobility.
specific application conditions than did
pesticides in use 40 years ago, today's farmers
observe their fields more closely than did
farmers 40 years ago.
(D) Certain pest-control methods that some farmers
use today do not involve the use of chemical
pesticides but are just as effective in eliminating
insect pests as those that do.
(E) Forty years ago, much less was known about the
effects of pesticides on humans and other
mammalian species than is now known.

62
9.Authorities in California required drivers to use their
headlights on a certain road during the daytime as 10. Which of the following, if true, most logically
well as at night and found that annual accident rates completes the passage?
on the road fell 15 percent from the previous level.
Every fusion reaction releases neutrinos. To test a
They concluded that applying the daytime rule
hypothesis about the frequency of fusion reactions in the
statewide would lead to a similar reduction in
Sun, physicists calculated the number of neutrinos the
accidents.
Sun would produce annually if the hypothesis were
Which of the following, if true, most strengthens the correct. From this they estimated how many neutrinos
authorities' argument? should pass through a particular location on Earth. The
(A) Because an alternate route became available, the fact that far fewer neutrinos were counted than were
volume of traffic on the test road decreased predicted to pass through the location would seem to
during the test period. prove that the hypothesis is wrong, except that------.
(B) Drivers were informed of the requirement to use (A) the physicists, using a different method for
their headlights on the test road by means of a estimating how many neutrinos should reach the
series of three conspicuous signs in each direction location, confirmed their original estimate
of travel. (B) there are several competing hypotheses about
(C) Under certain conditions, among them fog and the frequency of solar fusion reactions
heavy rain, most drivers in California already (C) there is not enough energy in the Sun to destroy
use their headlights during the daytime. a neutrino once it is released
(D) Full-scale application of the daytime rule would (D) the method used to count neutrinos detects no
cause headlight bulbs to burn out sooner than more than approximately ten percent of the
they currently do and thus to require more neutrinos that pass through
frequent replacement. (E) neutrinos released in the fusion reactions of
(E) The test road was selected to include a great other stars also reach the Earth
variety of the sorts of road conditions that
drivers in California are likely to encounter.

63
11.An economist concluded that Kregg Company
deliberately discriminated against people with a 12. Hastings’ contracture is a disorder of the connective
history of union affiliation in hiring workers for its tissue in one or both hands, most commonly causing
new plant. The economist's evidence is that, of the loss of mobility. A survey of thousands of
1,500 people hired to work at the new plant, only medical-insurance claims found that over 30 percent
100 had ever belonged to a labor union, whereas in of people who had one hand operated on for
Kregg Company's older plants, a much higher Hastings' contracture underwent surgery a second
proportion of workers have a history of union time for this disorder within three years. Clearly,
affiliation. therefore, a single surgical treatment of Hastings'
contracture is often ineffective at providing
Which of the following is an assumption on which long-term correction of the disorder.
the economist's argument depends?

(A) None of the people with a history of union


Which of the following, if true, most seriously weakens
affiliation who were hired to work at the new
the argument?
plant were union organizers.
(B) Applicants for jobs at the new plant were not (A) The medical-insurance claims did not specie
asked by Kregg's recruiters whether they had whether the surgery was on the patient's right or
ever belonged to a labor union. left hand.
(C) In the plants of some of Kregg's competitors, the (B) The surgical techniques used to treat Hastings'
workforce consists predominantly of union contracture are identical to those used
members. successfully to treat certain work-related
(D) The company believes that the cost of running injuries to the hand.
the new plant will be lower if labor unions are (C) A separate survey found that 90 percent of
not represented in the workforce. patients operated on for Hastings' contracture
(E) The pool of potential candidates for jobs at the report increased hand mobility within one
new plant included some people, in addition to month after the surgery.
those Kregg hired, with a history of union (D) All of the patients in the survey were required
affiliation. by their insurance companies to seek a second
opinion from a qualified surgeon before
undergoing the operation.
(E) Many people who have Hastings' contracture
choose to tolerate its effects rather than undergo
the risks of surgery.

64
14.A society can achieve a fair distribution of resources
13.The most widely used therapy for a certain type of only under conditions of economic growth. There
ulcer completely heals such ulcers in 44 percent of can be no economic growth unless the society
patients within six months. In a six-month trial of a guarantees equality of economic opportunity to all
new therapy for this type of ulcer. 80 percent of of its citizens. Equality of economic opportunity
ulcers treated achieved significant healing and 61 cannot be guaranteed unless a society's government
percent were completely heated. Since the trial actively works to bring it about.
treated only ulcers of this type that were worse than
average, the new therapy clearly promotes healing If the statements given are true, it can be properly
more effectively than the most widely used therapy. concluded from them that

(A) no government can achieve a fair distribution of


resources under conditions of economic growth
The answer to which of the following would be
(B) all societies that guarantee equality of economic
most useful in evaluating the argument given?
opportunity to all of their members are societies
(A) What differences are there, if any, in the ways that distribute resources fairly
that the two therapies are administered? (C) a society can achieve a fair distribution of
(B) Is there any significant difference between the resources only if its government actively works
costs associated with the two therapies? to bring about equality of economic opportunity
(C) What percentage of people with ulcers of this (D) there ran be no economic growth in a society
type who were treated with the most widely unless that society guarantees a fair distribution
used therapy for six months experienced of resources
significant healing? (E) some societies that experience economic growth
(D) How quickly do ulcers of this type, if left fail to guarantee equality of opportunity to all of
untreated, become significantly worse. their citizens
(E) What percentage of patients involved in the
six-month trial of the new therapy were
disappointed at the rate of healing were
experiencing?

65
15.High Towers, a company that occupies several office Which of the following, if true, most strengthens the
buildings, is considering installing new argument given?
energy-efficient lightbulbs in its buildings. The new
(A) If the new bulbs are widely adopted, as seems
bulbs require less than half the electricity consumed
likely, they will be produced in large enough
by the conventional bulbs currently used to produce
quantities to be offered at prices comparable to
the same amount of light. The new bulbs also last
those of conventional bulbs.
considerably longer. It follows that by replacing old
(B) The utility that supplies High Towers with
bulbs as they bum out with the new kind of bulb,
electricity offers discount rates to its largest
High Towers would significantly reduce its overall
customers.
lighting costs.
(C) High Towers has recently signed a contract to
occupy an additional small office building.
(D) High Towers has begun a campaign to
encourage its employees to turn off lights
whenever they leave a room.
(E) The company that manufactures the new bulbs
has been granted a patent on the innovative
technology used in the bulbs and thus has
exclusive rights to manufacture them.

66
练习 10

(在阅读套题训练之前勿做。

1. The painter Peter Brandon never dated his works, and 2. Dance critic from Europe: The improved quality of
their chronology is only now beginning to take shape ballet in the United States is the result of more
in the critical literature. A recent dating of a Brandon Europeans' teaching ballet in the United States than
self-portrait to 1930 is surely wrong. Brandon was 63 ever before. I know the proportion of teachers who
years old in 1930, yet the painting shows a young, were born and trained in Europe has gone up among
dark-haired man-obviously Brandon, but clearly not a ballet teachers in the United States, because last year,
man of 63. on my trip to New York, more of the ballet teachers I
met were from Europe-born and trained there -than
Which of the following, if justifiably assumed, allows ever before.
the conclusion to be properly drawn?
Which of the following identifies a questionable
(A) There is no securely dated self-portrait of
assumption made by the dance critic's reasoning?
Brandon that he painted when he was
significantly younger than 63. (A) The argument overlooks the possibility that some
(B) In refraining from dating his works, Brandon ballet teachers in the United States could have
intended to steer critical discussion of them away been born in Europe but trained in the United
from considerations of chronology. States.
(C) Until recently, there was very little critical (B) The argument assumes that the ballet teachers
literature on the works of Brandon. whom the critic met last year on the critic's trip to
(D) Brandon at age 63 would not have portrayed New York were a generally typical group of such
himself in a painting as he had looked when he teachers.
was a young man. (C) The argument assumes that the teaching of ballet
(E) Brandon painted several self-portraits that showed in the United States is superior to the teaching of
him as a man past the age of 60. ballet in Europe
(D) Other possible reasons for the improved mental
attitudes of United States dancers are not
examined.
(E) The argument assumes that dancers born and
trained in Europe are typically more talented than
dancers born and trained in the United States.

67
3.Mayor Four years ago when we reorganized the city
police department in order to save money, critics 4. It takes a particular talent to be a successful business
claimed that the reorganization would make the manager. Business courses can help people to solve
police less responsive to citizens and would thus lead management problems, but such courses can do so
to more crime. The police have compiled theft only for those people with managerial talent. Such
statistics from the years following the reorganization people should take business courses to acquire ideas
that show that the critics were wrong. There was an that they can subsequently use to good advantage if
overall decrease in reports of thefts of all kinds, management problems happen to arise.
including small thefts.
If the statements above are true, which of the following
must also be true on the basis of them?
Which of the following, if true, most seriously (A) People who are helped by business courses in
challenges the mayor's argument? solving management problems also have
(A) When city police are perceived as unresponsive, managerial talent.
victims of theft are less likely to report thefts to (B) People who are already skilled at solving
the police. management problems are unlikely to benefit
(B) The mayor's critics generally agree that police from business courses.
statistics concerning crime reports provide the (C) Most ideas that are used successfully in solving
most reliable available data on crime rates. management problems are those acquired in
(C) In other cities where police departments have business courses.
been similarly reorganized, the numbers of (D) People who lack managerial talent are more
reported thefts have generally risen following likely to take business courses than are people
reorganization. who have managerial talent.
(D) The mayor's reorganization of the police (E) Those people who have never taken business
department failed to save as much money as it courses are unable to solve management
was intended to save. problems when such problems arise.
(E) During the four years immediately preceding the
reorganization, reports of all types of theft had
been rising steadily in comparison to reports of
other crimes.

68
6. That sales can be increased by the presence of
5. When a driver is suspected of having had too much to sunlight within a store has been shown by the
drink, testing the driver's ability to walk a straight experience of the only Savefast department store
line gives a more reliable indication of fitness to with a large skylight. The skylight allows sunlight
drive than does testing the driver's blood-alcohol into half of the store, reducing the need for artificial
level. light. The rest of the store uses only artificial light.
Since the store opened two years ago, the
Which of the following, if true, best supports the claim departments on the sunlit side have had substantially
made in the statement above? higher sales than the other departments.
(A) Not all observers will agree whether or not an
Which of the following, if true, most strengthens the
individual has succeeded in walking a straight
argument?
line.
(B) Because of genetic differences and variations in (A) On particularly cloudy days, more artificial light
acquired tolerance to alcohol, some individuals is used to illuminate the part of the store under
suffer more serious motor impairment from a the skylight.
given high blood-alcohol level than do others. (B) When the store is open at night, the departments
(C) Tests designed to measure blood-alcohol levels in the part of the store under the skylight have
are accurate, inexpensive, and easy to sales that are no higher than those of other
administer. departments.
(D) More than half the drivers involved in fatal (C) Many customers purchase items from
accidents have blood-alcohol levels that exceed departments in both parts of the store on a single
the legal limit, whereas in less-serious accidents shopping trip.
the proportion of legally intoxicated drivers is (D) Besides the skylight, there are several
lower. significant architectural differences between the
(E) Some individuals with high blood-alcohol levels two parts of the store.
are capable of walking a straight line but are not (E) The departments in the part of the store under
capable of driving safely. the skylight are the departments that generally
have the highest sales in other stores in the
Savefast chain.

69
7. To protect beachfront buildings from ocean storms,
ocean resorts have built massive seawalls between 8. A study found that 70 percent of children surveyed in
beaches and the buildings. Not only do the seawalls 1970 had at one time had cavities, whereas only 50
block off some buildings' ocean view, but the percent of those surveyed in 1985 had ever had
beaches themselves become ever narrower, because cavities. The researchers concluded that the level of
sand can no longer creep inland as storms erode it at dental disease in children had declined between
the water's edge. 1970 and 1985.
If the information is correct, which of the following Which of the following, if true, would most
conclusions is most strongly supported on the basis seriously undermine the researchers' conclusion
of it? presented above?

(A) Since the ferocity of ocean storms is increasing, (A) Cavities are the most common kind of dental
increasingly high seawalls must be built disease to which children are subject.
between beaches and beachfront property. (B) The children surveyed came from a broad
(B) Even when beaches are heavily used by people, variety of income backgrounds.
they are necessary to the survival of the many (C) The children surveyed were selected from
wild species that use them. among students of teachers cooperating with the
(C) Seawalls constructed to protect beachfront researchers.
buildings will not themselves eventually be (D) The accuracy of cavity detection techniques has
damaged by storms and will not require, if they improved dramatically since 1970.
are to protect the buildings, expensive repair or (E) The children surveyed in 1985 were younger on
replacement. average than those surveyed in 1970.
(D) The conservation of beaches for future
generations should be the overriding goal of
shore management at ocean coasts.
(E) Trying to protect beachfront buildings by
constructing seawalls is counterproductive in
the long run for an oceanfront community
wishing to maintain itself as a beach resort.

70
9. David: Since attempting to preserve every species that
Which of the following is the main point of Karen's
is currently endangered is prohibitively expensive,
reply to David?
the endangered species whose value to humanity is
the greatest should be accorded the highest priority (A) Although it would be desirable to preserve all
for preservation. endangered species, doing so is not
Karen: Such a policy would he unsound because it economically feasible.
is impossible to predict the future value of a species, (B) Even if the value to humanity of a given species
nor is it always possible to assess the present value is known, that value should not be a factor in
of species whose contributions to humanity, though any decision on whether to expend effort to
significant, are indirect. preserve that species.
(C) Species whose contributions to humanity are
direct should have a higher priority for
preservation efforts than species whose
contributions to humanity are only indirect.
(D)Since the methods for deciding which species
have the most value to humanity are imperfect,
informed decisions cannot be made on the basis
of the assessment of such value.
(E) The preservation of endangered species whose
value to humanity can be reliably predicted is
more important than the preservation of species
whose value for humanity is unpredictable.

71
10. Roger: Reading a lot as a child causes
nearsightedness-difficulty seeing things at a 11. Years ago, consumers in Frieland began paying an
distance. energy tax in the form of two Frieland pennies for
Louise: I disagree. Any correlation between each unit of energy consumed that came from
near-sightedness and reading results from the fact nonrenewable sources. Following the introduction of
that children who have trouble seeing things at a this energy tax, there was a steady reduction in the
distance are likeliest to prefer those activities, such total yearly consumption of energy from
as reading, that involve looking at things close up. nonrenewable sources.

If the statements in the passage are true, then which of


Louise disputes Roger's claim by
the following must on the basis of them be true?
(A) demonstrating that an absurd conclusion would
follow if Roger's claim were accepted (A) There was a steady decline in the yearly revenues
(B) arguing that what Roger claims to be a cause of a generated by the energy tax in Frieland.
given phenomenon is actually its effect (B) There was a steady decline in the total amount of
(C) using an analogy to expose a flaw in Roger's energy consumed each year in Frieland.
reasoning (C)There was a steady increase in the use of
(D) pointing out that Roger's claim is renewable energy sources in Frieland
self-contradictory (D) The revenues generated by the energy tax were
(E) attempting to demonstrate that Roger uses the used to promote the use of energy from
term“nearsightedness” in an ambiguous way renewable sources.
(E) The use of renewable energy sources in Frieland
greatly increased relative to the use of
nonrenewable energy sources.

72
12. Despite a dramatic increase in the number of people
Which of the following, if true during the last three
riding bicycles for recreation in Parkville. a recent
years, best reconciles the apparent discrepancy in
report by the Parkville Department of Transportation
the facts above?
shows that the number of accidents involving
bicycles has decreased for the third consecutive (A) The Parkville Department of Recreation
year. confiscated abandoned bicycles and sold them at
auction to any interested Parkville residents.
(B) Increased automobile and bus traffic in Parkville
has been the leading cause of the most recent
increase in automobile accidents.
(C) Because of the local increase in the number of
people bicycling for recreation. many out -of
-town bicyclists ride in the Parkville area.
(D) The Parkville Police Department enforced
traffic rules for bicycle riders much more
vigorously and began requiring recreational
riders to pass a bicycle safety course.
(E) The Parkville Department of Transportation
canceled a program that required all bicycles to
be inspected and registered each year.

73
13. Do strong electric currents, by means of the 14. Neither the Sami nor the Kephrian delegations
electromagnetic fields that accompany them, cause attended the international conference. Beforehand. the
cancer in people who live and work nearby? delegations of Daqua and Kephria. allies whose
Telephone line workers. who work near such governments had grievances against Tessia. officially
currents every day, can provide a test case. They announced that one or both of the two would stay away
show elevated levels of brain cancer, therefore, the if the Tessian delegation attended the conference. In
hypothesis of electromagnetic causation is response, the Sami delegation officially announced that
supported. it would definitely attend if both the Daquan and
Kephrian delegations stayed away.
Which of the following if true, most seriously weakens
the argument? If the statements given are all true and all the
delegations adhered to their official announcements.
(A) Burying power lines and other measures to
it must also be true that the
protect the public from such electromagnetic
fields would be prohibitively expensive. (A) Daquan delegation attended the conference
(B) Telephone line workers are exposed to levels of (B) Daquan delegation did not attend the conference
chemical solvents high enough to cause brain (C) Sami government had no grievance against
cancer. Tessia
(C) High exposure to strong electromagnetic fields (D) Tessian delegation did not attend the conference
is correlated with a slightly higher-than-normal (E) Tessian delegation made no official
incidence of childhood leukemia, which is a announcement regarding its attendance at the
form of cancer. conference
(D) Public health officials who found that a group of
different illnesses in people living near a power
substation could not reliably be attributed to its
electromagnetic field were accused of covering
up the facts.
(E) Telephone line workers, like most people. have
electrical appliances at home, and most
electrical appliances, when turned on, are
surrounded by and electromagnetic field of
some measurable level.

74
15. On turning 65 years old, everyone living in the town
Which of the following is an assumption on which the
of Malton becomes eligible to receive a card that
argument depends?
guarantees discounts on most goods and services
sold in the town. Census records for 1990 show that (A) The town of Malton has no complete census
2, 450 inhabitants of Malton turned 64 in that year. records for 1991.
Yet . in 1991 over 3,000 people applied for and (B) The overall size of the population of Malton
properly received discount cards. So clearly some of grew by over 500 during 1990.
Malton's population growth between 1990 and 1992 (C) Fewer people applied for and received discount
must be attributable to migration into the city by cards in 1991 than did so in 1992.
people in their mid -60's (D) Among the people 65 years old or older who
moved into Malton in 1991. there was no one
who did not apply for a discount card .
(E) In general. people who applied for and received
discount cards in 1991 first became eligible to
do so in that year

75
答案

练习 1. EADED ECCBB DAA


练习 2. DEEED ADBAE CE
练习 3. BEDAE ECEEC BD
练习 4. ACBBE EAAEC DACB
练习 5. CBDDB EEBCA DB

练习 6. BCEED BDBEA D
练习 7. AACAE ADBBD BDC
练习 8. AEBDB AEDCD BADCB

练习 9. EDBBD BEBAE DDCCA


练习 10.DBAAB BEEDB ADBED

76

You might also like